Location via proxy:   [ UP ]  
[Report a bug]   [Manage cookies]                

OR Problems

Download as docx, pdf, or txt
Download as docx, pdf, or txt
You are on page 1of 104

Operation Research

Module 2. Linear programming problem

Lesson 3

MATHEMATICAL FORMULATION OF THE LINEAR PROGRAMMING PROBLEM AND ITS


GRAPHICAL SOLUTION

3.1 Introduction
A large number of business and economic situations are concerned with problems of planning and
allocation of resources to various activities. In each case there are limited resources at our disposal and our
problem is to make such a use of these resources so as to maximize production or to derive the maximum
profit, or to minimize the cost of production etc. Such problems are referred to as the problems of
constrained optimization. Linear programming (LP) is one of the most versatile, popular and widely used
quantitative techniques. Linear Programming is a technique for determining an optimum schedule chosen
from a large number of possible decisions. The technique is applicable to problem characterized by the
presence of a number of decision variables, each of which can assume values within a certain range and
affect their decision variables. The variables represent some physical or economic quantities which are of
interest to the decision maker and whose domain are governed by a number of practical limitations or
constraints which may be due to availability of resources like men, machine, material or money or may be
due quality constraint or may arise from a variety of other reasons. The most important feature of linear
programming is presence of linearity in the problem. The word Linear stands for indicating that all
relationships involved in a particular problem are linear. Programming is just another word for
�planning� and refers to the process of determining a particular plan of action from amongst several
alternatives. The problem thus reduces to maximizing or minimizing a linear function subject to a number
of linear inequalities
3.2 Definitions of Various Terms Involved in Linear Programming
3.2.1 Linear
The word linear is used to describe the relationship among two or more variables which are directly
proportional. For example, if the production of a product is proportionately increased, the profit also
increases proportionately, then it is a linear relationship. A linear form is meant a mathematical expression
of the type,
����������� 

 
3.2.2 Programming

jayaramulu123@gmail.com Page 1
Operation Research
The term �Programming� refers to planning of activities in a manner that achieves some optimal result
with resource restrictions. A programme is optimal if it maximizes or minimizes some measure or criterion
of effectiveness, such as profit, cost or sales.
3.2.3 Decision variables and their relationship
The decision (activity) variables refer to candidates (products, services, projects etc.) that are competing
with one another for sharing the given limited resources. These variables are usually inter-related in terms
of utilisation of resources and need simultaneous solutions. The relationship among these variables should
be linear.
3.2.4 Objective function
The Linear Programming problem must have a well defined objective function for optimization. For
example, maximization  of profits or minimization of costs or total elapsed time of the system being
studied. It should be expressed as linear function of decision variables.
3.2.5 Constraints
There are always limitations on the resources which are to be allocated among various competing
activities. These resources may be production capacity, manpower, time, space or machinery. These must
be capable of being expressed as linear equalities or inequalities in terms of decision variables.
3.2.6 Alternative courses of action
There must be alternative courses of action. For example, there may be many processes open to a firm for
producing a commodity and one process can be substituted for another.
3.2.7 Non-negativity restriction
All the variables must assume non-negative values, that is, all variables must take on values equal to or
greater than zero. Therefore, the problem should not result in negative values for the variables.
3.2.8 Linearity and divisibility
All relationships (objective functions and constraints) must exhibit linearity, that is, relationships among
decision variables must be directly proportional. For example, if our resources increase by some
percentage, then it should increase the outcome by the same percentage. Divisibility means that the
variables are not limited to integers. It is assumed that decision variables are continuous, i.e., fractional
values of these variables must be permissible in obtaining an optimal solution.
3.2.9 Deterministic
In LP model (objective functions and constraints), it is assumed that the entire model coefficients are
completely known (deterministic), e.g. profit per unit of each product, and amount of resources available
are assumed to be fixed during the planning period.
3.3 Formulation of a Linear Programming Problem  

jayaramulu123@gmail.com Page 2
Operation Research
 The formulation of the Linear Programming Problem (LPP) as mathematical model involves the following
key steps:
Step 1. Identify the decision variables to be determined and express them in terms of algebraic symbols
as X1,X2, --- , Xn.
Step 2. Identify the objective which is to be optimized (maximized or minimized) and express it as a
linear function of the above defined decision variables.
Step 3. Identify all the constraints in the given problem and then express them as linear equations or
inequalities in terms of above defined decision variables.
Step 4. Non-negativity restrictions on decision variables.
The formulation of a linear programming problem can be illustrated through the following examples:
Example 1
A milk plant manufactures two types of products A and B and sells them at a profit of Rs. 5 on type A and
Rs. 3 on type B. Each product is processed on two machines G and H. Type A requires one minute of
processing time on G and two minutes on H; type B requires one minute on G and one minute on H. The
machine G is available for not more than 6 hours 40 minutes, while machine B is available for 8 hours 20
minutes during any working day; formulate the problem as LP problem.
Solution
Let X1 be the number of products of type A and X 2 the number of products of type B. Since the profit on
type A is Rs. 5 per product, 5X1 will be the profit on selling X1units of type A. Similarly, 3X2 will be the
profit on selling X2 units of type B. Therefore, total profit on selling X 1 units of A and X2 units of B is
given by Z = 5X1+3X2. Since this has to be maximized, hence the objective function can be expressed as
 Max Z = 5X1+3X2   (Objective function)

Since machine G takes 1 minute time on type A and 1 minute time on type B, the total time in minutes
required on machine G is given by X1+X2. Similarly the total time in minutes required on machine H is
given by 2X1+X2. But machine G is not available for more than 6 hour 40 minutes (400 minutes), therefore
 X1+X2 ≤ 400      (first constraint on machine G)

Also the machine H is available for 8 hours 20 minutes only, therefore


 2X1+X2 ≤ 500    (second constraint on machine H)
Since it is not possible to produce negative quantities,
 X1 ≥0, X2 ≥0      (non-negativity restrictions)
Example 2
A dairy plant packs two types of milk in pouches viz., full cream and single toned. There are sufficient
ingredients to make 20,000 pouches of full cream & 40,000 pouches of single toned. But there are only

jayaramulu123@gmail.com Page 3
Operation Research
45,000 pouches into which either of the products can be put. Further it takes three hours to prepare enough
material to fill 1000 pouches of full cream milk & one hour for 1000 pouches of single toned milk and
there are 66 hours available for this operation. Profit is Rs. 8 per pouch for full cream milk and Rs. 7 per
pouch for single toned milk. Formulate it as a linear programming problem.
Solution
Let number of full cream and single toned milk pouches to be packed is X1 and X2

  Let profit be Z

  Objective function:        Max. Z = 8X1+7X2

  Subjecttoconstraints:  

X1 +X2 ≤ 45000 (1)

 X1 ≤ 20000 (2)

 X2 ≤ 40000 (3)

   (4)

Non-negativity restrictions  X1, X2 ≥0

Example 3
Consider two different types of food stuffs say F1 and F2. Assume that these food stuffs contain vitamin
A and B. Minimum daily requirements of vitamin A and B are 40mg and 50mg respectively. Suppose food
stuff F1 contains 2mg of vitamin A and 5mg of vitamin B while F 2 contains 4mg of vitamin A and 2mg of
vitamin B. Cost per unit of F1 is Rs. 3 and that of F2 is Rs. 2.5. Formulate the minimum cost diet that would
supply the body at least the minimum requirements of each vitamin.

Solution
Let number of units needed for food stuffs F1 and F2 to meet the daily requirements of vitamins A and B be
respectively X1 and X2..
      Objective function: Minimize Z =3X1+2.5X2
            Subject to constraints:                  
2X1+4X2 ≥ 40
 5X1+2X2 ≥50
       Non-negativity restrictions       X1, X2 ≥ 0
3.4 Mathematical Formulation of a General Linear Programming Problem

jayaramulu123@gmail.com Page 4
Operation Research
The general formulation of LP problem can be stated as follows. If we have n-decision variables X 1, X2,�,
Xn and m constraints in the problem , then we would have the following type of mathematical formulation
of LP problem

Optimize (Maximize or Minimize) the objective function:

Z=C1X1 +C2X2 + + CnXn (Eq.3.4.1)

Subject to satisfaction of m- constraints:

Where the constraint may be in the form of an inequality (≤ or ≥) or even in the form of an equality (=) and
finally satisfy the non-negativity restrictions
X1,X2,,Xn ≥0              (Eq.3.4.3)
where Cj (j=1, 2,�.,n); bi (i=1, 2,�,m) and aij are all constants and m<n, and the decision variables X j ≥0,
j=1, 2,�,n. If bi is the available amount of resource i then aij is amount of resource i that must be allocated
(technical coefficient) to each unit of activity j.
NOTE: By convention, the values of RHS parameters bi (i=1, 2, 3,�,m) are restricted to non-negative
values only. If any value of bi is negative then it is to be changed to a positive value by multiplying both
sides of the constraint by -1. This not only changes the sign of all LHS Coefficients and of RHS parameters
but also changes the direction of inequality sign.
 
3.4.1 Matrix form of LP problem
The linear programming problem (3.4.1), (3.4.2) and (3.4.3) can be expressed in matrix form as
Maximize Z = CX                                 (objective function )
 Subject to AX = b, b ≥ 0                      (constraint equation )
 X ≥ 0                                      (non-negativity restriction )
 Where X = (X1,X2, � , Xn), C = (C1,C2, � , Cn) and b = (b1,b2, � , bm)

��������������� 

3.5 Graphical Solution of Linear Programming Problem

jayaramulu123@gmail.com Page 5
Operation Research
LP problems which involve only two variables can be solved graphically. Such a solution by geometric
method involving only two variables is important as it gives insight into more general case with any
number of variables.
3.5.1 Feasible solution
A set value of the variables of a linear programming problem which satisfies the set of constraints and the
non-negative restrictions is called feasible solution of the problem.
3.5.2 Feasible region
The collection of all feasible solutions is known as the feasible region. Any point which does not lie in the
feasible region cannot be a feasible solution to the LP problems. The feasible region does not depend on
the form of the objective function in any way. If we can represent the relations of the general LP problem
on an n dimensional space, we will obtain a shaded solid figure (known as Convex-polyhedron)
representing the domain of the feasible solution.
3.5.3 Optimal solution
A feasible solution of a linear programming problem which optimizes its objective function is called the
optimal solution of the problem. Theoretically, it can be shown that objective function of a LP problem
assumes its optimal value at one of the vertices (called extreme points) of this solid figure.
Steps to find graphical solution of the linear programming problem
Step 1: Formulate the linear programming problem.
Step 2: Draw the constraint equations on XY-plane.
Step 3: Identify the feasible region which satisfies all the constraints simultaneously. For less than or
equal to constraints the region is generally below the lines and for greater than or equal to constraints,
the region is above the lines.
Step 4:  Locate the solution points on the feasible region. These points always occur at the vertices of
the feasible region.
Step 5: Evaluate the optimum value of the objective function.
The geometric interpretation and solution for the LP problem using graphical method is illustrated with the
help of following examples
Example 4 Find the graphical solution of problem formulated in example 1.
Max Z = 5X1+3X2
Subject to
 X1+X2 ≤ 400
 2X1+X2 ≤ 500
 X1 ≥ 0 , X2 ≥0
Solution

jayaramulu123@gmail.com Page 6
Operation Research
Any point lying in the first quadrant has X 1, X2 ≥ 0 and hence satisfies the non-negativity restrictions.
Therefore, any point which is a feasible solution must lie in the first quadrant. In order to find the set of
points in the first quadrant which satisfy the constraints, we must interpret geometrically inequalities such
as X1+X2≤ 400. If equality holds then we have the equation X 1+X2= 400 i.e., any point on the straight line
satisfies the equation. Any point lying on or below the line X1+X2= 400 satisfies the constraint X1+X2≤
400. However, no point lying above the line satisfies the inequality. In a similar manner, we can find the
set of points satisfying 2X1+X2 ≤ 500 and the non-negativity restrictions are all the points in the first
quadrant.  The set of points satisfying the constraints and the non-negativity restrictions is the set of points
in the shaded region of the figure OABC as shown in Fig 3.1 Any point in this region is a feasible solution,
and only the points in this region are feasible solutions. To solve the LP problem, we must find the point or
points in the region of feasible solutions which give the largest value of the objective function. Now for
any fixed value of Z, Z = 5X1+3X2   is a straight line, for each different values of Z, we obtain a different
line. Again, all the lines corresponding to different values of Z are parallel; clearly our interest is to find the
line with the largest value of Z which has at least one point in common with the region of feasible
solutions. The line Z= 5X1+3X2 is drawn for various values of Z. It can be seen that the point farthest from
Z = 5X1+3X2   and yet in the feasible region is B (100, 300) and thus point B maximizes Z while satisfying
all the constraints. In other words, the corner B of the region of feasible solutions is the optimal solution of
the LP problem. Since B is the intersection of the lines X 1+X2=400 and 2X1+X2=500, it can be seen that at
vertex B, X1=100 and X2=300. The maximum value of Z = 5(100)+3(300)=1400.The fact that the optimum
occurred at the vertex B of the feasible region is not a coincidence but on the other hand represents a
significant property of optimal solutions of all LP problems. To find the optimal solution find the values of
objective function at the various extreme points as shown in the following Table 3.1
    Table 3.1 Computation of maximum value of objective function
Extreme Point Coordinates Profit Function Z = 5X1+3X2
O X1=0 , X2=0 Z=5(0)+3(0)=0
A X1=0 , X2=400 Z=5(0)+3(400)=1200
B X1=100, X2=300 Z=5(100)+3(300)=1400
C X1=250, X2=0 Z=5(250)+3(0)=1250
Note: A fundamental theorem in LP states that the feasible region of any LP is a convex polygon (that is
the n dimensional version of two dimensional polygon), with a finite number of vertices, and further for
any LP problem, there is at least one vertex which provides an optimal solution. Whenever a LP problem
has more than one optimal solution, we say that there are alternative optimal solutions. Physically, this
means that the resources can be combined in more than one way to maximize profit.

jayaramulu123@gmail.com Page 7
Operation Research

Fig. 3.1 Feasible region 


Example 5 Find the graphical solution of problem formulated in Example 2.
Solution
Let number of full cream and single toned milk pouches to be produced is X1and X2 and Let profit be Z

 Objective function:        Max. Z = 8X1+7X2

 Subject to constraints:   X1 + X2   45000             (1)

 X1   20000                     (2)

X2   40000                      (3)

 3X1 + X2   66000            (4)

 Non-negativity restrictions  X1, X2 0
First of all draw the graphs of these inequalities (as discussed in Example 5) which is shown in Fig. 3.2.

jayaramulu123@gmail.com Page 8
Operation Research

Fig. 3.2 Feasible region


To find the optimal solution find the values of objective function at the various extreme points as shown in
the following Table 3.2
    Table 3.2 Computation of maximum value of objective function
Extreme Point Coordinates Profit Function Z = 8X1+7X2
O X1=0 , X2=0 Z=8(0)+7(0)=0
A X1=0 , X2=40000 Z=8(0)+7(40000)=280000
B X1=5000, X2=40000 Z=8(5000)+7(40000)=320000
C X1=10500, X2=34500 Z=8(10500)+7(34500)=325500
D X1=20000 , X2=6000 Z=8(20000)+7(6000)=202000
E X1=20000 , X2=0 Z=8(20000)+7(0)=160000
 
So maximum value of Z occurs at point C (10500, 34500) so it is the optimal solution. It can be concluded
that Dairy Plant must produce 10500 pouches of full cream milk and 34500 pouches of single toned milk.
Let us consider the following example to illustrate graphical solution for minimization problem
3.5.4 Special cases in linear programming

jayaramulu123@gmail.com Page 9
Operation Research
Up till now we have discussed those problems which have a unique optimal solution. However, it is
possible for LP problem to have following special cases.
3.5.4.1 Unbounded solution
A linear  programming problem is considered to have an unbounded solution if it has no limits on the
constraints and further, the common feasible region is not bounded in any respect.
Example 6 Find the graphical solution of problem formulated in example 3.
Solution
Let number of units needed for food stuffs F1 and F2 to meet the daily requirements of vitamins A and B be
respectively X1 and X2..
      Objective function: Minimize Z = 3X1+2.5X2
            Subject to constraints:  2X1+4X2 ≥ 40
5X1+2X2 ≥50
 Non-negativity restrictions:  X1, X2 0
First of all draw the graphs of these inequalities (as discussed in example 5) .Since the inequalities are of
the greater than or equal to type, the feasible region is formed by considering the area to the upper right
side of each equation i.e. away from origin .The shaded area which is shown above CBD is satisfied by the
two constraints as shown in Fig. 3.3 is feasible region. The feasible region for minimization problem is
unbounded and unlimited. Since the optimal solution corresponds to one of the corner (extreme) points, we
will calculate the values of objective function for each corner point viz. D (20, 0); B (7.5, 6.25) and C (0,
25). The calculations are shown in Table 3.3
Table 3.3 Table showing the computation of minimum value of objective function
Extreme Point Coordinates Cost Function  Min. Z = 3X1+2.5X2
D X1=20 , X2=0 Z=3(20)+2.5(0)=60
B X1=7.5 , X2=6.25 Z=3(7.5)+2.5(6.25)=38.125
C X1=0, X2=25 Z=3(0)+2.5(25)=62.5
 

jayaramulu123@gmail.com Page 10
Operation Research

Fig. 3.3 Feasible region for minimization problem


The minimum cost is obtained at the corner point B(7.5,6.25) i.e. X 1=7.5 and X2=6.25. Hence to minimize
the cost and to meet the daily requirements of vitamin A and B, number of units needed of food stuffs
F1 and F2 be 7.5 and 6.25 respectively.
3.5.5  Infeasible solution
In this there is no solution to an LP Problem that satisfies all the constraints. Graphically, it means that no
feasible solution region exists. Such a condition indicates that the LP problem has been wrongly
formulated.
3.5.6  Redundant constraint
In a properly formulated LP problem, each of the constraints will define a portion of the boundary of the
feasible solution region. Whenever, a constraint does not define a portion of the boundary of the feasible
solution region, it is called a redundant constraint. Let us consider the following example to illustrate this.
Example 7 Maximize Z=1170X1+1110X2
Subject to:
9X1+5X2 ≥ 500
7X1+9X2 ≥ 300
5X1+3X2 ≤ 1500
jayaramulu123@gmail.com Page 11
Operation Research
7X1+9X2 ≤1900
2X1+4X2 ≤ 1000
X1, X2 ≥ 0
The feasible region of this LP problem is indicated in Fig. 2.4. In this the feasible region has been
formulated by two constraints.
 9X1+5X2 ≥ 500
 7X1+9X2 ≤1900
 X1, X2 ≥ 0
 

Fig 3.4 Feasible Region and Redundant Constraints


The remaining three constraints, although present, is not affecting the feasible region in any manner. Such
constraints are known as redundant constraints.

Module 2. Linear programming problem

Lesson 4
SIMPLEX METHOD
4.1 Introduction
In the previous chapter we considered the formulation of linear programming problems and the graphic
method of solving them. Although the graphical approach to the solution of such problems is an invaluable
aid to understand its basic structure, the method is of limited application in industrial problems as the
number of variables occurring there is often considerably large. The Simplex Method provides an efficient
technique which can be applied for solving linear programming problems of any magnitude-involving two
or more decision variables. The Simplex Method is the name given to the solution algorithm for solving LP
jayaramulu123@gmail.com Page 12
Operation Research
problems developed by George B. Dantzig in 1947. It is iterative procedure having fixed computational
rules that leads to a solution to the problem in a finite number of steps. By using this one is capable of
solving large LP problems efficiently.

4.2 Principle of Simplex Method


As the fundamental theorem of LP problem tells us that at least one basic feasible solution of any LP
problem must be optimal, provided the optimal solution of the LP problem exists. Also, the number of
basic feasible solutions of the LP problem is finite and at the most nCm (where, n is number of decision
variables and m is the number of constraints in the problem). On the other hand, the feasible solution may
be infinite in number. So it is rather impossible to search for optimal solutions from amongst all feasible
solutions. Furthermore, a great labour will also be required in finding out all the basic feasible solutions
and select that one which optimizes the objective function. In order to remove this difficulty; a simple
method was developed by Dantzig (1947) which is known as Simplex Algorithm. Simplex Algorithm is a
systematic and efficient procedure for finding corner point solutions and taking them for optimality. The
evaluation of corner points always starts from the point of origin. This solution is then tested for optimality
i.e. it tests whether an improvement in the objective function is possible by moving to adjacent corner point
of the feasible function space. This iterative search for a better corner point is repeated until an optimal
solution if it exists, is determined.  
4.3  Basic Terms Involved in Simplex Procedure
The following terms relevant for solving a linear programming problem through simplex procedure are
given below:
4.3.1  Standard form of linear programming problem
The standard form of LP problem is to develop the procedure for solving general LP problem. The optimal
solution of the standard form of a LP problem is the same as original LP problem. The characteristics of
standard form are given in following steps:

Step 1.     All the constraints should be converted to equations except for the non-negativity restrictions
which remain as inequalities (≥0).
Step 2.     The right side element of each constraint should be made non-negative.
Step 3.     All variables must have non-negative values.
Step 4.     The objective function should be of maximization form.
4.3.2 Slack variables
If a constraint has less than or equal sign, then in order to make it on equality we have to add something
positive to the left hand side. The non-negative variable which is added to the left hand side of the
constraint to convert it into equation is called the slack variable. For example, consider the constraints.
����������� 3X1 + 5X2 ≤ 2, 7X1 + 4X2 ≤ 5, X1, X2 ≥ 0                
We add the slack variables S1 ≥ 0, S2 ≥ 0 on the left hand sides of above inequalities respectively to obtain

jayaramulu123@gmail.com Page 13
Operation Research
����������� 3X1+5X2+S1 = 2
����������� 7X1+4X2+S2 = 5
����������� X1, X2, S1, S2 ≥ 0
4.3.3 Surplus variables
If a constraint has greater than or equal to sign, then in order to make it an equality we have to subtract
something non-negative from its left hand side. The positive variable which is subtracted from the left hand
side of the constraint to convert it into equation is called the surplus variable.
For example, consider the constraints.
����������� 3X1 + 5X2 ≥ 2, 2X1 + 4X2 ≥ 5, X1, X2 ≥0      
We subtract the surplus variables S3 ≥0, S4 ≥ 0 on the left hand sides of above inequalities respectively to
obtain
����������� 3X1+5X2 -S3 = 2
����������� 2X1 + 4X2 -S4 = 5
����������� X1, X2, X3, X4 ≥0
4.3.4 Solution to LPP
Any set X = {X1, X2, X3,�, Xn+m} of variables is called a solution to LP problem, if it satisfies the set of
constraints only.
4.3.5 Feasible solution (FS)
Any set X = {X1, X2, X3,��,Xn+m} of variables is called a feasible solution of L.P. problem, if it satisfies
the set of constraints as  well as non-negativity restrictions.
4.3.6 Basic solution (BS)
For a system of m simultaneous linear equations in n variables (n>m), a solution obtained by setting (n-m)
variables equal to zero and solving for the remaining variables is called a basic solution. Such m variables
(of course, some of them may be zero) are called basic variables and remaining (n-m) zero-valued variables
are called non-basic variables.
4.3.7 Basic feasible solution (BFS)
A basic feasible solution is a basic solution which also satisfies the non-negativity restrictions, that is all
basic variables are non-negative. Basic solutions are of two types
(a)    Non-degenerate BFS: A non-degenerate basic feasible solution is the basic feasible solution which
has exactly m positive Xj (j=1, 2,�,m). In other words, all basic m variables are positive, and the
remaining (n �m) variables will be all zero.
(b)   Degenerate BFS: A basic feasible solution is called degenerate, if one or more basic variables are
zero-valued.
4.3.8 Optimum basic feasible solution

jayaramulu123@gmail.com Page 14
Operation Research
A basic feasible solution is said to be optimum, if it also optimizes (maximizes or minimizes) the objective
function.
4.3.9 Unbound solution
If the value of the objective function Z can be increased or decreased indefinitely, such solutions are called
unbounded solutions.
4.4 Computational Aspect of Simplex Method for Maximization Problem
Step 1: Formulate the linear programming model. If we have n-decision variables X1, X2, Xn and m
constraints in the problem , then mathematical formulation of L P problem is
Maximize Z=C1X1+ C2X2+�+CnXn                                  
Subject to the constraints:
����������� a11 X1+ a12 X2+ � +a1n Xn ≤ b1
����������� a21 X1+ a22 X2� + � + a2n Xn ≤ b2
����������������������������������������� �
����������� am1 X1+ am2 X2� + � + amn Xn ≤ bm
����������� X1, X2, � Xn� ≥ 0
Step 2: Express the mathematical model of LP problem in the standard form by adding slack variables in
the left�hand side of the constraints and assign zero coefficient to these variables in the
objective function.Thus we can restate the problem in terms of equations as follows:
Maximize Z=C1X1+
C2X2 +�+ CnXn  +0Xn+1  +0Xn+2+�+0Xn+m                           ���������������������������������������
��������������������������������������������

Subject to the constraints:


a11 X1+ a12 X2+ �+ a1n Xn +Xn+1  = b1
a21 X1+ a22 X2+�+ a2n Xn +Xn+2  = b2
.
am1 X1+ am2X2+�+amn Xn +Xn+m = bm
 
����������� X1, X2, � ,Xn,���� Xn+1,Xn+2 ,�, Xn+m ≥0
 
Step 3: Design the initial feasible solution .An initial basic feasible solution is obtained by setting 
X1=X2=� =Xn =0. Thus, we get Xn+1=b1, Xn+2 =b2 ,�, Xn+m =bm.
Step 4: Construct the starting (initial) simplex tableau. For computational efficiency and simplicity, the
initial basic feasible solution, the constraints of the standard LP problem as well as the objective function
can be displayed in a tabular form, called the Simplex Tableau as shown below.
Table 4.1  Initial simplex tableau
Cj (contribution per unit) → c1 c2 � cn 0 0 � 0 Minimum
Cb Basic Value of Coefficient Matrix Identify Matrix Ratio*

jayaramulu123@gmail.com Page 15
Operation Research
Variables Basic
Variables
  B b(=XB) X1 X2 � Xn Xn+1 Xn+2 � Xn+m
0 s1 b1 a11 a12 � a1n 1 0 � 0  
0 s2 b2 a21 a22 � a2n 0 1 � 0
. . . . . � .        
. . . . . � .        
. . . . . � .        
0 sm bm am1 am2 � a mn 0 0 � 1
Contribution 0 0 � 0 0 0 � 0  
loss per unit:
Net c1 c2 � cn 0 0 � 0
contribution
per units:
 
* Negative ratio is not to be considered.
 
The interpretation of the data in the above �Tableau� is given as under. Other simplex tableau will have
similar interpretations.
(i)    The first row, called the objective row of the simplex table indicates the values of Cj (j subscript
refer to the column number) which are the coefficients of the (m + n) variables in the objective
function. These coefficients are obtained directly from the objective function and the
value Cj would remain the same in the succeeding tables. The second row of the table provides the
major column headings for the table and these column headings remain unchanged in the
succeeding tables of the Simplex Method.
(ii)  The first column labelled CB, also known as objective column, lists the coefficient of the current
basic variables in the objective function. The second column labelled �Basic variables� points
out the basic variables in the basis, and in the initial simplex tableau these basic variables are the
slack variables. The third column labelled �Solution values� (= xB), indicates the resources or the
solution values of the basic variables.
(iii) The body matrix (under non-basic variables) in the initial simplex tableau consists of the
coefficients of the decision variables in the constraint set.
(iv)The identity matrix in the initial simplex tableau represents the coefficient of the slack variables that
have been added to the original inequalities to make them equation. The matrix under non-basic
variables in the simplex tableau is called coefficient matrix. Each simplex tableau contains an
identity matrix under the basic variables.
(v)   To find an entry in the Zj row under a column, we multiply the entries of that column by the
corresponding entries of CB � column and add the products, i.e., Z =  .
The Zj entry under the �Solution column� gives the current value of the objective function.
(vi) The final row labeled ∆j = Zj - Cj called the index (or net evaluation) row, is used to determine
whether or not the current solution is optimal. The calculation of Zj - Cj row simply involves

jayaramulu123@gmail.com Page 16
Operation Research
subtracting each Zj value from the corresponding Cj value for that column, which is written at the
top of that column. Each entry in the ∆j� row represents the net contribution (or net marginal
improvement) to the objective function that results by introducing one unit of each of the respective
column variables.
Step 5: Test the Solution for Optimality. Examine the index row of the above simplex tableau. If all the
elements in the index row are positive then the current solution is optimal. If there exist some negative
values, the current solution can further be improved by removing one basic variable from the basis and
replacing it by some non-basic one.
Step 6: Revision of the Current Simplex Tableau. At each iteration, the Simplex Method moves from the
current basic feasible solution to a better basic feasible solution. This involves replacing one current basic
variable (called the departing variable) by a new non-basic variable (called the entering variable).

(a)    Determine which variable to enter into the solution-mix net. One way of doing this is by
identifying the column (and hence the variable) with the most negative number in the ∆ j row of
the previous table.

(b)    Determine the departing variable or variable to be replaced. Next we proceed to determine


which variable must be removed from the basis to pave way for the entering variable. This is
accomplished by dividing each number in the quantity (or solution values) column by the
corresponding number in the pivot column selected in (a), i.e., we compute the respective ratios
b1/a1j, b2/a2j, �, bm/amj (only for those aij�s; i=1,2,�, m which are strictly positive). These
quotients are written in the last column labelled �Minimum Ratio� of the simplex tableau.
The row corresponding to smallest of these non-negative ratios is called the pivot (or key) row
and the corresponding basic variable will leave the basis. Let the minimum of { b 1/a1j, b2/a2j,
�, bm/amj ; aij > 0} be bk/akj , then corresponding variables in the pivot row sk will be termed as
outgoing (or departing) variable in the next tableau to be constructed just after we put an arrow
→ of type to right of kth row of the simplex tableau 1.

(c)     Identify the pivot number. The non-zero positive number that lies at the intersection of the
pivot column and pivot row of the given table is called the pivot (or key) number. We place a
circle around the number.
Step 7: Evaluate (update) the new solution. After identifying the entering and departing variable, find the
new basic feasible solution by constructing a new simplex tableau from the current one by using the
following steps:
(a)    Compute new values for the pivot row by simply dividing every element of the pivot row by
the pivot number.
(b)   New entries in the CB column and XB column are entered in the new table of the current
solution
(c)    Compute new values for each of the remaining rows by using the following formula
New row numbers=Number in old rows-{(corresponding number above or below pivot number)
x (corresponding number in the row replaced in (a))}
jayaramulu123@gmail.com Page 17
Operation Research
(d)    Test for optimality. Compute the Zj and index rows as previously demonstrated in the initial
simplex tableau. If all numbers in the index row are either zero or positive, an optimal solution
has been made attained. i.e., there is no variable which can be introduced in the solution to
cause the objective function value to increase.
4.      Revise the solution. If any of the numbers in the index ( ∆j = Zj - Cj) row are negative, repeat the entire
steps 5 & 6 again until an optimal solution has been obtained.

The above procedure is illustrated through the following example.

Example 1
A firm produces three products A, B, and C each of which passes through three different departments
fabrication, finishing, packaging. Each unit of product A requires 3, 4 and 2 hours respectively, B requires
5, 4 and 4 hours respectively and C requires 2, 4 and 5 hours respectively in 3 departments respectively.
Every day 60 hours are available in fabrication department, 72 hours in finishing and 100 hours in
packaging department. If unit contribution of unit A is Rs. 5, Rs. 10 for B and Rs. 3 for C. Then determine
number of units of each product so that total contribution to cost is maximized and also determine if any
capacity would remain unutilized.
Solution:        
Step 1: Formulate this as LPP.  Let X1, X2 and X3 be the number of units produced of the products A, B and
C respectively.

����������� Objective function:        Max Z = 5X1 + 10X2 + 3X3

����������� Subject to constraints:    3X1 + 5X2 + 2X3 ≤ 60

                                ����������� 4X1 + 4X2 + 4X3 ≤ 72

����������������������������������� ���������
���� 2X1 + 4X2 + 5X3 ≤ 100         X1, X2, X3 ≥ 0

Step 2: Now converting into standard form of LPP

                  Max Z= 5X1 + 10X2 + 3X3 + 0S1 + 0S2 + 0S3

                  3X1 + 5X2 + 2X3 + S1 = 60

                  4X1 + 4X2 + 4X3 + S2 = 72

                  2X1 + 4X2 + 5X3 + S3 = 100                   X1, X2, X3, S1, S2, S3 ≥ 0

                  where S1 ,S2 and S3 are slack variables.

Step 3: Find the initial feasible solution .An initial basic feasible solution is obtained by setting   X1 =0,
X2 = 0 and X3 = 0. Thus, we get S1 = 60, S2 = 72 and S3 =100.
jayaramulu123@gmail.com Page 18
Operation Research
Step 4: Construct the starting (initial) simplex tableau.
      Cj  → 5 10 8 0 0 0  
  B.V. CB XB X1 X2 X3 S1 S2 S3 Minimum
Ratio
R1 S1 0 60 3 (5) 2 1 0 0 60/5=12

R2 S2 0 72 4 4 4 0 1 0 72/4=18
R3 S3 0 100 2 4 5 0 0 1 100/4=25
      Zj 0 0 0 0 0 0  
      −5 −10 −8 0 0 0  
                 

Step 5: The most negative value of ∆j is −10 hence X2 is the incoming variable (↑) and the least positive
minimum ratio is 12 hence S1 is the outgoing variable (⟶). The element under column X2 and row R1   is

the key element i.e. 5 so divide each element of row R 1 by 5 (i.e.  ). Subtract appropriate multiples
of this new row from the remaining rows, so as to obtain zeros in the remaining positions. Performing the
row operations 
we get the second Simplex tableau as   
         
      Cj → 5 10 8 0 0 0  
  B.V. CB XB X1 X2 X3 S1 S2 S3 M.R.
X2 10 12 3/5 1 2/5 1/5 0 0 12/2/5=30
S2 0 24 8/5 0 12/5 −4/5 1 0 24/12/5=10
S3 0 52 -2/5 0 17/5 −4/5 0 1 52/17/5=15.294
      Zj 6 10 4 2 0 0  
      1 0 −4 2 0 0  
 
Step 6:  The most negative value of ∆j is -4 hence X3 is the incoming variable (↑) and the least positive
minimum ratio is 10 hence S2 is the outgoing variable (�). The element under column X2 and row
�R1   is the key element i.e. 5 so divide each element of row R 2b by 12/5 (i.e. Rc → Rb * 5/12). Subtract
appropriate multiples of this new row from the remaining rows, so as to obtain zeros in the remaining

positions. Performing the row operations  .


We get the third Simplex tableau as
    Cj  → 5 10 8 0 0 0
B.V. CB XB X1 X2 X3 S1 S2 S3
X2 10 8 1/3 1 0 1/3 −1/6 0

jayaramulu123@gmail.com Page 19
Operation Research
X3 8 10 2/3 0 1 −1/3 5/12 0
S3 0 18 −8/3 0 0 1/3 −17/12 1
    Zj 26/3 10 8 2/3 5/3 0
    j
11/3 0 0 2/3 5/3 0

It is apparent from this table that all ∆ j =Zj − Cj are positive and therefore an optimum solutions
is reached . So X1 = 0, X2 = 8, X3 = 10

����������� Z = 5X1 + 10X2 + 8X3 = 160

And  also as S3 is coming out to be 18 so there are 18 hours unutilized in finishing department.
In case the objective function of the given LP problem is to be minimized, then we convert it into a
problem of maximization by using
Min. Z* = − Max. (−Z). The procedure of finding optimal solution using Simplex Method is illustrated
through the following example:
Example 4.2  
Minimize the objective function Z: X1 � 3X2 + 2X3
Subject to constraints    3X1 � X2 + 3 X3 ≤ 7
             ������ −2X1 + 4X2 ≤12
         � −4X1 + 3X2 + 8X3 ≤10
                       X1, X2, X3 ≥0
Solution      
Converting this minimization problem into maximization problem

Objective function Max Z#: −X1 � 3X2 + 2X3 + 0S1 + 0S2 + 0S3

Constraints 3X1 � X2 + 3 X3 + S1 = 7

����������� −2X1 + 4X2 + S2 = 12

����������� −4X1 + 3X2 + 8X3 + S3 = 10

����������� X1, X2, X3, S1, S2, S3 ≥0

Starting Simplex Table


    Cj −1 3 −2 0 0 0  
B.V. CB XB X1 X2 X3 S1 S2 S3 Min. Ratio
S1 0 7 3 −1 3 1 0 0 -
S2 0 12 −2 4 1 0 1 0          3    →
S3 0 10 −4 3 8 0 0 1 10/3
    1 −3 2 0 0 0  
j

 
jayaramulu123@gmail.com Page 20
Operation Research

Now performing the row operations  �� 


    Cj −1 3 −2 0 0 0 Min.
Ratio
B.V. CB XB X1 X2 X3 S1 S2 S3  
S1 0 10 5/2 0 3 1 1/4 0      4  →
X2 3 3 −1/2 1 0 0 1/4 0 -
X3 0 1 −5/2 0 8 0 −3/4 1 -
    −1/2 0 2 0 3/4 0  
j

Now performing the row operations  � 


↑   Cj −1 3 −2 0 0 0
B.V. CB XB X1 X2 X3 S1 S2 S3
X1 −1 4 1 0 6/5 2/5 1/10 0
X2 3 5 0 1 3/5 1/5 3/10 0
S3 0 11 0 0 11 1 −1/2 1
    ∆j 0 0 13/5 1/5 4/5 0
 

Now all ∆j are positive. Therefore, Optimal Solution is reached  

Therefore, X1 = 4, X2 = 5 & X3 = 0 & Max Z# = −1 4 + 3 5 = 11 Or Minimum Z = −Z# = −11

Module 3. Transportation problem

Lesson 6
INITIAL BASIC FEASIBLE SOLUTION
6.1 Introduction
In the last lesson we have learnt about Transportation Problem (TP) and its formulation. Transportation
problem can be solved by simplex method and transportation method. In simplex method the solution is
very lengthy and cumbersome process because of the involvement of a large number of decision and
artificial variables.  In this lesson we will look for an alternate solution procedure called transportation
method in which initial basic feasible solution of a TP can be obtained in a better way by exploiting the
special structure of the problem.
6.2 Some Definitions
The following terms are to be defined with reference to Transportation Problem

6.2.1 Feasible solution (FS)

jayaramulu123@gmail.com Page 21
Operation Research
By feasible solution we mean a set of non-negative individual allocations (Xij ≥ 0) which satisfies the row
and column conditions (rim condition).
6.2.2 Basic feasible solution (BFS)
A feasible solution is said to be basic if the number of positive allocations equals m+n-1; that is one less
than the number of rows and columns in a transportation problem.
6.2.3 Optimal solution
A feasible solution (not necessarily basic) is said to be optimal if it minimizes the total transportation cost.
6.3 Solution for Transportation Problem
The solution algorithm to a transportation problem can be summarized into following steps:
Step 1: Formulate the problem. The formulation of transportation problem is similar to a LP problem
formulation. Here the objective function is to minimize the total transportation cost and the
constraints are the supply and demand available at each source and destination, respectively.
Step 2: Obtain an initial basic feasible solution. This initial basic feasible solution can be obtained by using
any of the following five methods:
a)      North West Corner Rule
b)      Minimum cost method
c)      Row Minimum Method
d)     Column Minimum Method
e)      Vogel�s Approximation Method
The solution obtained by any of the above methods must fulfill the following conditions:
        i.      The solution must be feasible, i.e., it must satisfy all the supply and demand constraints. This is

called rim condition.
      ii.      The number of positive allocations must be equal to m+n�1, where, m is number of rows and n

is number of columns.
The solution that satisfies both the above mentioned conditions is called a non-degenerate basic feasible
solution.
Step 3: Test the initial solution for optimality. Using any of the following methods one can test the
optimality of an initial basic solution:
i.��� Stepping Stone Method
ii.�� Modified Distribution Method (MODI)
If the solution is optimal then stop, otherwise, find a new improved solution.
Step 4: Updating the solution. Repeat Step 3 until the optimal solution is obtained.
6.4 Methods of Obtaining an Initial Basic Feasible Solution
Five methods are described to obtain the initial basic feasible solution of the transportation problem. These
methods can be explained by considering the following example
Example 1

jayaramulu123@gmail.com Page 22
Operation Research
Let us consider the formulation of TP as given in example 1 of the previous lesson which can be
represented by the following e transportation table:
 

 
Each cell in the table represents the amount transported from one route to one chilling center. The amount
placed in each cell is, therefore, the value of a decision. The smaller box within each cell contains the unit
transportation cost for that route.
6.4.1 North west corner rule (NWC) method
Step I: The first assignment is made in the cell occupying the upper left-hand (North West) corner of the
transportation table. The maximum feasible amount is allocated there. That is X 11 = Min (a1, b1)
and this value of X11 is then entered in the cell (1, 1) of the transportation table.
Step II: a) If b1>a1, we move down vertically to the second row and make the second allocation of
magnitude X21=Min. (a2, b1- X11) in the cell (2, 1).
�b)   If b1< a1� we move horizontally to the second column and make the second allocation of
magnitude X12 = Min. (a1�X11, b2) in the cell (1, 2).
�c)   If b1= a1, there is a tie for the second allocation. One can make the second allocation of
magnitude X12 = Min. (a1�a1, b1) = 0 in the cell (1, 2) or X21=Min. (a2, b1�b1) = 0 in the cell
(2, 1)
Step III: Repeat steps I & II by moving down towards the lower right corner of the transportation table
until all the rim requirements are satisfied.
6.4.1.1 Illustration of north �west corner rule
Let us illustrate the above method with the help of example 1given above. Following North-West corner
rule, the first allocation is made in the cell (1,1), the magnitude being X 11=Min.(150,140)=140. The second
allocation is made in cell (1,2) and the magnitude of allocation is given by X 12=Min. (150-140,120)=10.
Third allocation is made in the cell (2,2), the magnitude being X 22=Min. (160,120-10)=110. The magnitude
of fourth allocation in the cell (2,3) is given by X 23=Min. (160-110, 90)=50. The fifth allocation is made in
the cell (3,3), the magnitude being X33= Min. (90-50, 90)=40. The sixth allocation in the cell (3,4) is given
by X34=Min. (90-40, 50) = 50. Now all requirements have been satisfied and hence an initial basic feasible
solution to T.P. has been obtained.

jayaramulu123@gmail.com Page 23
Operation Research

 The transportation cost according to the above allocation is given by z = 16 � 140 + 18 � 10 + 19 � 110
+ 14 � 50 + 15 � 40 + 10 � 50 = 6310
6.4.2 Matrix minima ( Least cost entry) method
Various steps of Matrix Minima method are given below:
Step I: Determine the smallest cost in the cost matrix of the transportation table. Let it be Cij. Allocate Xij =
Min. (ai, bj) in the cell (i, j).
Step II:� a) If Xij = ai, cross off the ith row of the transportation table and decrease bi by ai and go to Step
III.
              b) If Xij = bj, cross off the jth column of the transportation table and decrease ai by bj and go to Step
III.
              c) If Xij = ai = bj, cross of either the ith row or the jth column but not the both.
Step III: Repeat steps I & II for the resulting reduced transportation table until all the requirements are
satisfied. Whenever the minimum cost is not unique, make an arbitrary choice among the
minima.
6.4.2.1 Illustration of matrix minima ( Least cost entry) method
Let us illustrate this method by considering example 1given earlier in this lesson . The first allocation is
made in the cell (3, 4) where the cost of transportation is minimum, the magnitude being X 34=50. This
satisfies the requirement of S chilling centre. Therefore cross off the fourth column. The second allocation
is made in the cell (3,2) having minimum cost among remaining cells and its  magnitude being X32=Min.
(120,90-50)=40. This satisfies the supply of route C. Cross off the third row. Among the remaining cells,
the minimum cost is found in cell (2, 3) so the third allocation is done in cell (2, 3) and its magnitude being
X23=Min.(90,160)=90. The requirement of the chilling center R is fulfilled, hence third column is crossed
off. Out of the remaining cells the minimum cost is found in cell (1,1) and its magnitude is
X11=Min(140,150)=140, as the requirement of chilling center  P is exhausted hence column first is deleted.
Among remaining two cells minimum cost is found in cell (1,2) and its magnitude X 12=Min(80,150-
140)=10 and the last allocation is done in the cell (2,2) and its magnitude is X 22=Min(80-10,70)=70. Now
all requirements have been satisfied and hence an initial basic feasible solution to T.P. has been obtained
and given in the following table.
jayaramulu123@gmail.com Page 24
Operation Research

The transportation cost according to the above allocation is given by z = 16 � 140 + 18 � 10 + 19 � 70 +


11 � 40 + 14 � 90 + 10 � 50 = 5950

6.4.3 Row minima method


Various steps of Row Minima method are given below:
Step I: The smallest cost in the first row of the transportation table is determined, let it be Cij. Allocate the
maximum feasible amount Xij = Min. (ai, bj) in the cell (i, j), so that either the capacity of
origin Oi is exhausted or the requirement at destination Dj is satisfied or both.
Step II: a) If Xij = a1, so that the availability at origin O1 is completely exhausted, cross off the first row of
the table and move down to the second row.
  b) If Xij=bj so that the requirement at destination Dj is satisfied. Cross off the jth column and
reconsider the first row with the remaining availability of origin O1.
� c) If Xij=a1=bj , the origin capacity of O1 is completely exhausted as well as the requirement at
destination D1 is completely satisfied. An arbitrary choice is made. Cross off the first row and
make the second allocation X1k = 0 in the cell (1, k) with C1k being the new column cost in the
first row. Cross off the first row and move down to the second row.
Step III: Repeat steps I & II for the resulting reduced transportation problem until the requirements are
satisfied.
6.4.3.1 Illustration of row minima method
 Let us illustrate this method by considering Example 1given earlier in this lesson. In the first row smallest
cost is in the first row which is in the cell (1,4) allocate minimum possible amount X14=Min(50,150)=50.
This exhausts requirement of S chilling centre and thus we cross off the fourth column from the
transportation table. In the resulting transportation table, the minimum cost in the first row is 16 in the cell
(1, 1), the second allocation is made in the cell (1, 1) with X 11=Min (140,150-50) =100. This satisfies the
supply of Route A, hence first row is deleted. In the second row of the remaining transportation table, the
minimum cost is present in cell (2,3) so third allocation is done X 23=Min(90,160)=90.This satisfies the
requirement of R chilling centre, hence third column is deleted. The next allocation is made in the cell (2,1)
with X21=Min(140-100,160-90)=40.This exhausts the requirement of Route P and so we cross-out the first
jayaramulu123@gmail.com Page 25
Operation Research
column .The fifth allocation is made in cell X22=Min(120,70-40)=30.This exhausts the supply of route B
and hence it is crossed out.  In third row the cell left is (3, 2). Hence the last allocation of amount X 32=Min
(120-30, 90) =90 is obviously made in the cell (3,2). Now all requirements have been satisfied and hence
an initial basic feasible solution of TP has been obtained and given in the following table.

The transportation cost according to the above allocation is given by� z = 16 � 100 + 12 � 50 + 17 �


40 + 19 � 30 + 14 � 90 + 11 � 90 = 5700

6.4.4 The column minima method

This method is same as Row minima method except that we apply the concept of minimum cost in column
instead of row.

6.4.5 Vogel�s approximation method (VAM)


The Vogel�s Approximate Method takes into account not only the least cost Cij but also the costs that just
exceed Cij. Various steps of the method are given below:
Step I: For each row of the transportation table identify the smallest and the next to smallest cost.
Determine the difference between them for each row. These are called penalties (opportunity cost).
Put them along side of the transportation table by enclosing them in parenthesis against the
respective rows. Similarly compute these penalties for each column.
Step II: Identify the row or column with the largest penalty among all the rows and columns. If a tie
occurs, use any arbitrary tie breaking choice. Let the greatest penalty correspond to ith row and
let cij be the smallest cost in the ith row. Allocate the largest possible amount Xij = min. (ai, bj) in
the (i, j) of the cell and cross off the ith row or jth column in the usual manner.
Step III: Re-compute the column and row penalty for the reduced transportation table and go to step II.
Repeat the procedure until all the requirements are satisfied.
Remarks:
1.         A row or column �difference� indicates the minimum unit penalty incurred by failing to make an
allocation to the smallest cost cell in that row or column.

jayaramulu123@gmail.com Page 26
Operation Research
2.         It will be seen that VAM determines an initial basic feasible solution which is very close to the
optimum solution that is the number of iterations required to reach optimum solution is minimum in
this case.
 
6.4.6 Illustration of vogel�s approximation method (VAM)

Let us illustrate this method by considering Example 1 discussed before in this lesson .

For each row and column of the transportation table determine the penalties and put them along side of the
transportation table by enclosing them in parenthesis against the respective rows and beneath the
corresponding columns. Select the row or column with the largest penalty i.e. (7) (marked with an arrow)
associated with second column and allocate the maximum possible amount to the cell (3,2) with minimum
cost and allocate an amount X32=Min(120,90)=90 to it. This exhausts the capacity of route C. Therefore,
cross off third row. The first reduced penalty table will be:

In the first reduced penalty table the maximum penalty of rows and columns occurs in column 3, allocate
the maximum possible amount to the cell (2,3) with minimum cost and allocate an amount

jayaramulu123@gmail.com Page 27
Operation Research
X23=Min(90,160)=90 to it . This exhausts the capacity of chilling center R .As such cross off third column
to get second reduced penalty table as given below.

In the second reduced penalty table there is a tie in the maximum penalty between first and second row.
Choose the first row and allocate the maximum possible amount to the cell (1,4) with minimum cost and
allocate an amount X14=Min(50,150)=50 to it . This exhausts the capacity of chilling center S so cross off
fourth column to get third reduced penalty table as given below:

The largest of the penalty in the third reduced penalty table is (2) and is associated with first row and
second row. We choose the first row arbitrarily whose min. cost is C11 = 16. The fourth allocation of
X11=min. (140, 100) =100 is made in cell (1, 1). Cross off the first row. In the fourth reduced penalty table
i.e.  second row, minimum cost occurs in cell (2,1) followed by cell (2,2)  hence allocate X21=40 and
X22=30. Hence the whole allocation is as under:

jayaramulu123@gmail.com Page 28
Operation Research
The transportation cost according to the above allocation is given by z = 16 � 100 + 12 � 50 + 17 � 40 +
19 � 30 + 14 � 90 + 11 � 90 = 5700

Note: Generally, Vogel�s Approximation Method is preferred over the other methods because the initial
BFS obtained is either optimal or very close to the optimal solution.

Module 3. Transportation problem

Lesson 7
OPTIMAL SOLUTION

7.1 Introduction
After examining the initial basic feasible solution, the next step is to test the optimality of basic feasible
solution. Though the solution obtained by Vogel�s method is not optimal, yet the procedure by which it
was obtained often yields close to an optimal solution. So to say, we move from one basic feasible solution
to a better basic feasible solution, ultimately yielding the minimum cost of transportation. There are two
methods of testing optimality of a basic feasible solution. The first of these is called the Stepping Stone
method and the second method is called Modified Distribution method (MODI).By applying either of these
methods, if the solution is found to be optimal, then problem is solved. If the solution is not optimal, then a
new and better basic feasible solution is obtained. It is done by exchanging a non-basic variable for one
basic variable i.e. rearrangement is made by transferring units from an occupied cell to an empty cell that
has the largest opportunity cost and then shifting the units from other related cells so that all the rim
requirements are satisfied.                            

7.2 Some Definitions
7.2.1 Non-degenerate solution
A basic feasible solution of an m x n transportation problem is said to be non-degenerate, if it has the
following two properties:
(1)   Starting BFS must contain exactly (m + n -1) number of individual allocations.
(2)   These allocations must be in independent positions.
Here by independent positions of a set of allocations we mean that it is always impossible to form closed
loops through these allocations. The following table show the non-independent and independent positions
indicated by the following diagram:
 

jayaramulu123@gmail.com Page 29
Operation Research

7.2.2 Degeneracy
If the feasible solution of a transportation problem with m origins and n destinations has fewer than m+n-1
positive Xij (occupied cells), the problem is said to be a degenerate transportation problem. Degeneracy can
occur at two stages:
a)      At the initial stage of Basic Feasible Solution.
b)      During the testing of the optimal solution.
To resolve degeneracy, we make use of artificial quantity.
7.2.3 Closed path or loop
This is a sequence of cells in the transportation tableau such that
a)      each pair of consecutive cells lie in either the same row or the same column.
b)      no three consecutive cells lie in the same row or column.
c)      the first and last cells of a sequence lie in the same row or column.
d)     no cell appears more than once in the sequence.
7.3 Stepping Stone Method
In this method, the net cost change can be obtained by introducing any of the non-basic variables
(unoccupied cells) into the solution. For each such cell find out as to what effect on the total cost would be
if one unit is assigned to this cell. The criterion for making a re-allocation is simply to know the desired
effect upon various costs. The net cost change is determined by listing the unit costs associated with each
cell and then summing over the path to find the net effect. Signs are alternate from positive (+) to negative
(-) depending upon whether shipments are being added or subtracted at a given point. A negative sign on
the net cost change indicates that a cost reduction can be made by making the change while a positive sign
will indicate a cost increase. The stepping stone method for testing the optimality can be summarized in the
following steps:
Steps  
1.   Determine an initial basic feasible solution.
2.   Make sure that the number of occupied cells is exactly equal to m+n-1, where m is number of    rows
and n is number of columns.

3.   Evaluate the cost�effectiveness of transporting goods through the transportation routes not currently
in solution. The testing of each unoccupied cell is conducted by following four steps given as under:
jayaramulu123@gmail.com Page 30
Operation Research
a.    Select an unoccupied cell, where transportation should be made. Beginning with this cell, trace a
closed path using the most direct route through at least three occupied cells and moving with
only horizontal and vertical moves. Further, since only the cells at the turning points are
considered to be on the closed path, both unoccupied and occupied boxes may be skipped over.
The cells at the turning points are called Stepping Stone on the path.
b.   Assign plus (+) and minus (-) sign alternatively on each corner cell of the closed path traced
starting a plus sign at the unoccupied cell to be evaluated.
c.    Compute the net change in the cost along the closed path by adding together the unit cost figures
found in each cell containing a plus sign and then subtracting the unit cost in each square
containing the minus sign.
d.   Repeat sub step (a) through sub step (b) until net change in cost has been calculated for all
unoccupied cells of the transportation table. 
4.   Check the sign in each of the net changes .If all net changes computed are greater than or equal to
zero, an optimal solution has been reached. If not, it is possible to improve the current solution and
decrease total transportation cost.
5.   Select the unoccupied cell having the highest negative net cost change and determine the maximum
number of units that can be assigned to a cell marked with a minus sign on the closed path,
corresponding to this cell. Add this number to the unoccupied cell and to other cells on the path
marked with a plus sign. Subtract the number from cells on the closed path marked with a minus sign.
6.   Go to step 2 and repeat the procedure until we get an optimal solution.
To demonstrate the application of this method let us take example 1 discussed in lesson 6 for consideration.
Example 1 Find optimal solution of the problem given in example 1 of lesson 6 by stepping stone method.
Solution
The initial basic feasible solution using Least Cost Method was found in example 1 of previous lesson with
the following allocations. 

The Transportation cost according to the above allocation is given by


��������������� 

jayaramulu123@gmail.com Page 31
Operation Research
Beginning at first unoccupied cell (A,Q) trace a closed path  and this closed path is
AR ⟶ BR ⟶ BQ ⟶ AQ.  Assign plus (+) and minus (-) sign alternatively on each corner cell of the
closed path traced starting a plus sign at the unoccupied cell .Evaluate  the cost�effectiveness of
transporting milk  on different transportation routes of each unoccupied cell and given in the following
table:

Unoccupied cell Closed Path Net Cost change (in Rs.)


(A,R) 21-14+19-18= 8
(A,S) 12-10+11-18= -5
(B,P) 17-16+18-19= 0
(B,S) 13-10+11-19= -5
 (C,P) 32-16+18-11= 23
(C,R) 15-11+19-14= 9
Select the unoccupied cell having the highest negative net cost change i.e. cell (A,S).  The maximum
number of units that can be transported to a cell marked with a minus sign on the closed path is 10. Add
this number to the unoccupied cell and to other cells on the path marked with a plus sign. Subtract the
number from cells on the closed path marked with a minus sign. New solution is given in the following
table.

The transportation cost according to the above allocation is given by


��������������� 

Beginning at the first unoccupied cell (A,Q) and repeating the steps given above. Evaluate the
cost�effectiveness of transporting milk on different transportation routes of each unoccupied cell given in
the following table:

jayaramulu123@gmail.com Page 32
Operation Research
��������������� 

Unoccupied cell Closed Path Net Cost change (in Rs.)


(A,Q) 18-12+10-11=  5
(A,R) 21-12+10-11+19-14= 13
(B,P) 17-19+11-10+12-16= -5
(B,S) 13-10+11-19= -5
 (C,P) 32-16+12-10= 18
(C,R) 15-11+19-14= 9
Select the unoccupied cell having the highest negative net cost change i.e. cell (B,S) .  The maximum
number of units that can be transported to a cell marked with a minus sign on the closed path is 40. Add
this number to the unoccupied cell and to other cells on the path marked with a plus sign. Subtract the
number from cells on the closed path marked with a minus sign. New solution is given in the following
table.

The transportation cost according to the above allocation is given by


��������������� 

Beginning at the first unoccupied cell (A,Q) and repeating the steps given above. Evaluate the
cost�effectiveness of transporting milk on different transportation routes of each unoccupied cell and
given in the following table:
Unoccupied cell Closed Path Net Cost change (in Rs.)
(A,Q) 18-12+13-19=  0
(A,R) 21-12+13-14=  8
(B,P) 17-16+12-13=  0

jayaramulu123@gmail.com Page 33
Operation Research
(C,P) 32-16+12-13+14-11= 18
(C,R) 15-11+19-14= 9
(C,S) 10-13+19-11= 5
 
Since all the unoccupied cells have positive values for the net cost change, hence optimal solution has
reached and optimal cost is Rs. 5700.
7.4 MODI (Modified Distribution) Method
The MODI (Modified Distribution) method is an efficient method of testing the optimality of a
transportation solution. In stepping stone method each of the empty cells is evaluated for the opportunity
cost by drawing a closed loop. In situations where a large number of sources and destinations are involved,
this would be a very time consuming exercise. This method avoids this kind of extensive scanning and
reduces the number of steps required in the evaluation of the empty cells. This method allows us to
compute improvement indices quickly for each unused square without drawing all of the closed paths.
Because of this, it can often provide considerable time savings over other methods for solving
transportation problems. It provides new means of finding the unused route with the largest negative
improvement index. Once the largest index is identified, we are required to trace only one closed path. This
path helps determine the maximum number of units that can be transported via the best unused route. Steps
involved for finding out the optimal solution of transportation problem are as follows:
1. � Determine an initial basic feasible solution using any one of the five
methods discussed earlier. We start with a basic feasible solution
consisting of (m + n � 1) allocations in independent positions.
2. Determine a set of (m +n) numbers u r (r = 1, 2, 3, . . . , m) and v s (s = 1, 2, 3,
. . . ,n), such that, for each occupied cell (r, s)         
3.
Compute the opportunity cost using    .
4. Check the sign of each opportunity cost. If the opportunity costs of all the
unoccupied cells are either positive or zero, the given solution is the
optimum solution. On the other hand, if one or more unoccupied cell has
negative opportunity cost, the given solution is not an optimum solution and
further savings in transportation cost are possible.
5. Select the unoccupied cell with the smallest negative opportunity cost as the
cell to be included in the next solution.
6. Draw a closed path or loop for the unoccupied cell selected in the previous
step, using the most direct route through at least three occupied cells and
moving with only horizontal and vertical moves.
7. Assign alternate plus and minus signs at the unoccupied cells on the corner
points of the closed path with a plus sign at the cell being evaluated.
jayaramulu123@gmail.com Page 34
Operation Research
8. Determine the maximum number of units that should be transported to this
unoccupied cell. The smallest value with a negative position on the closed
path indicates the number of units that can be transported to the entering
cell. Now, add this quantity to all the cells on the corner points of the closed
path marked with plus signs and subtract it from those cells marked with
minus signs. In this way an unoccupied cell becomes an occupied cell.

9. Repeat the whole procedure until an optimum solution is obtained.


Let us illustrate this method by taking example 1 into consideration.

Example 2 Find optimal solution of the problem given in example 1 of lesson 6 by MODI method.
Solution
The initial basic feasible solution using Least Cost Method was found in example 1 of previous lesson with
the following allocations. 

The transportation cost according to the above allocation is given by


����������� 
Step 1: Since the number of occupied cells are m+n-1=3+4-1=6 the initial solution is non �degenerate.
Thus optimal solution can be obtained.
Step 2: We first set up an equation for each occupied cell:

jayaramulu123@gmail.com Page 35
Operation Research
����������������� 
Step 3:  After the row and column numbers have been computed, the next step is to evaluate the
opportunity cost for each of the unoccupied cells by using the relationship

����������������� 
Unoccupied cell Opportunity Cost
(A,R) d13 = 21 � (0+13)= 8
(A,S) d14 = 12- (0+7) = -5
(B,P) d21 = 17 � (1+16) = 0
 (B,S) d24 = 13 - (1+17) = -5
(C,P) d31 = 32 � (-7+16) = 23
(C,R) d33 = 15 � (-7+13) = 9
Step 4: Select the unoccupied cell having the highest negative net cost change i.e. cell (A,S).  The
maximum number of units that can be transported to a cell marked with a minus sign on the closed path is
10. Add this number to the unoccupied cell and to other cells on the path marked with a plus sign. Subtract
the number from cells on the closed path marked with a minus sign. New solution is given in the following
table

The transportation cost according to the above allocation is given by

To test this solution for further improvement, we recalculate the values of ur and vs based on the second
solution for each of the occupied cells.

jayaramulu123@gmail.com Page 36
Operation Research
 
Again, we again calculate the opportunity costs for each unoccupied cell by using the relationship
�������������        
 
Unoccupied cell Opportunity Cost
(A,Q) d12 = 18-(0+13) = 5
(A,R) d13 = 21-(0+8) = 13
(B,P) d21 = 17 � (6+16) = -5
 (B,S) d24 =13 � (6+12) = -5
(C,P) d31 = 32-(-2+16) = 18
(C,R) d33 = 15 � (-2+8) = 9
Since the opportunity cost in the cell (B,P) and (B,S) are negative , the current basic feasible solution is not
optimal and can be improved . Choosing the unoccupied cell (B,S), we trace a closed path that begins and
ends at this cell. The maximum number of units that can be transported to a cell marked with a minus sign
on the closed path is 40. Add this number to the unoccupied cell and to other cells on the path marked with
a plus sign. Subtract the number from cells on the closed path marked with a minus sign. New solution is
given in the following table.

To test this solution for further improvement, we recalculate the values of ur and vs based on the second
solution for each of the occupied cells.

 
Again, we calculate the opportunity costs for each unoccupied cell by using the relationship
jayaramulu123@gmail.com Page 37
Operation Research
�������������   
 
Unoccupied cell Opportunity Cost
(A,Q) d12 = 18-(0+18) = 0
(A,R) d13 = 21-(0+13) = 8
(B,P) d21 = 17 � (1+16) = 0
(C,P) d31 = 32-(-7+16) = 23
(C,R) d32 = 15 � (-7+13) = 9
(C,S) d34 = 10 � (-7+12) = 5
In the above table the opportunity costs for each unoccupied cell is positive value, we conclude that
solution is optimal one and the transportation cost
is ����������� 

7.5 Maximization in a Transportation Problem


Although the transportation problems which were dealt so far were of minimization, we may also come
across of transportation problems with maximization objectives. When origins are related to destinations
by a profit function instead of cost, the objective is to be maximize instead of minimize. The most
convenient way to deal with the maximization case is to transform the profits to relative costs which are
carried out by subtracting the profit associated with each cell from the largest profit in the matrix. Once the
optimal solution for the minimization problem is obtained, the value of objective function is determined
with reference to the original profit matrix.
                                                                              
                                                     
 Module 4. Assignment problem
Lesson 9
SOLUTION OF ASSIGNMENT PROBLEM
9.1 Introduction

Although assignment problem can be solved either by using the techniques of Linear Programming or by
the transportation method yet the assignment method developed by D. Konig, a Hungarian mathematician
known as the Hungarian method of assignment problem is much faster and efficient. In order to use this
method, one needs to know only the cost of making all the possible assignments. Each assignment problem
has a matrix (table) associated with it. Normally, the objects (or people) one wishes to assign are expressed
in rows, whereas the columns represent the tasks (or things) assigned to them. The number in the table
would then be the costs associated with each particular assignment. It may be noted that the assignment
problem is a variation of transportation problem with two characteristics firstly the cost matrix is a square
matrix and secondly the optimum solution for the problem would be such that there would be only one
assignment in a row or column of the cost matrix.
9.2 Solution of Assignment Problem
The assignment problem can be solved by the following four methods:
jayaramulu123@gmail.com Page 38
Operation Research
a)      Complete enumeration method
b)      Simplex Method
c)      Transportation method
d)      Hungarian method
9.2.1 Complete enumeration method
In this method, a list of all possible assignments among the given resources and activities is prepared. Then an
assignment involving the minimum cost, time or distance or maximum profits is selected. If two or more
assignments have the same minimum cost, time or distance, the problem has multiple optimal solutions. This
method can be used only if the number of assignments is less. It becomes unsuitable for manual calculations if
number of assignments is large
9.2.2 Simplex method
This can be solved as a linear programming problem as discussed in section 8.1.3 of the last lesson and as
such can be solved by the simplex algorithm.
9.2.3 Transportation method
As assignment is a special case of transportation problem, it can also be solved using transportation model
discussed in module 3. The solution obtained by applying this method would be degenerate. This is
because the optimality test in the transportation method requires that there must be m+n-1= (2n-1) basic
variables. For an assignment problem of order n x n there would be only n basic variables in the solution
because here n assignments are required to be made. This degeneracy problem of solution makes the
transportation method computationally inefficient for solving the assignment problem.
9.2.4 Hungarian assignment method
The Hungarian method of assignment provides us with an efficient means of finding the optimal solution.
The Hungarian method is based upon the following principles:
(i)     If a constant is added to every element of a row and/or column of the cost matrix of an assignment
problem the resulting assignment problem has the same optimum solution as the original problem
or vice versa.
(ii)   The solution having zero total cost is considered as optimum solution.        
Hungarian method of assignment problem (minimization case) can be summarized in the following steps:
Step I:   Subtract the minimum cost of each row of the cost (effectiveness) matrix from all the elements of
the respective row so as to get first reduced matrix.
Step II:   Similarly subtract the minimum cost of each column of the cost matrix from all the elements of
the respective column of the first reduced matrix. This is first modified matrix.
Step III:   Starting with row 1 of the first modified matrix, examine the rows one by one until a row
containing exactly single zero elements is found. Make any assignment by making that zero in or enclose
the zero inside a. Then cross (X) all  other zeros in the column in which the assignment was made. This
eliminates the possibility of making further assignments in that column.
jayaramulu123@gmail.com Page 39
Operation Research
Step IV: When the set of rows have been completely examined, an identical procedure is applied
successively to columns that is examine columns one by one until a column containing exactly single zero
element is found. Then make an experimental assignment in that position and cross other zeros in the row
in which the assignment has been made.
Step V: Continue these successive operations on rows and columns until all zeros have been either
assigned or crossed out and there is exactly one assignment in each row and in each column. In such case
optimal assignment for the given problem is obtained.
Step VI: There may be some rows (or columns) without assignment i.e. the total number of marked zeros
is less than the order of the matrix. In such case proceed to step VII.
Step VII: Draw the least possible number of horizontal and vertical lines to cover all zeros of the starting
table. This can be done as follows:
1.      Mark (√) in the rows in which assignments has not been made.
2.      Mark column with (√) which have zeros in the marked rows.
3.      Mark rows with (√) which contains assignment in the marked column.
4.      Repeat 2 and 3 until the chain of marking is completed.
5.      Draw straight lines through marked columns.
6.      Draw straight lines through unmarked rows.
By this way we draw the minimum number of horizontal and vertical lines necessary to cover all zeros at
least once. It should, however, be observed that in all n x n matrices less than n lines will cover the zeros
only when there is no solution among them. Conversely, if the minimum number of lines is n, there is a
solution.
Step VIII: In this step, we
1.    � Select the smallest element, say X, among all the not covered by any of the lines of the table;
and
2.     Subtract this value X from all of the elements in the matrix not covered by lines and add X to
all those elements that lie at the intersection of the horizontal and vertical lines, thus
obtaining the second modified cost matrix.
Step IX: Repeat Steps IV, V and VI until we get the number of lines equal to the order of matrix I, till an
optimum solution is attained.
Step X: We now have exactly one encircled zero in each row and each column of the cost matrix. The
assignment schedule corresponding to these zeros is the optimum assignment. The above technique is
explained by taking the following examples
Example 1
A plant manager has four subordinates, and four tasks to be performed. The subordinates differ in
efficiency and the tasks differ in their intrinsic difficulty. This estimate of the times each man would take to
perform each task is given in the effectiveness matrix below.

  I II III IV
A 8 26 17 11

jayaramulu123@gmail.com Page 40
Operation Research
B 13 28 4 26
C 38 19 18 15
D 19 26 24 10

How should the tasks be allocated, one to a man, so as to minimize the total man hours?
Solution
Step I : Subtracting the smallest element in each row from every element in that row, we get the first
reduced matrix.
 
0 18 9 3
9 24 0 22
23 4 3 0
9 16 14 0
 
Step II: Next, we subtract the smallest element in each column from every element in that column; we get
the second reduced matrix.
Step III: Now we test whether it is possible to make an assignment using only zero distances.
0 14 9 3
9 20 0 22
23 0 3 0
9 12 14 0
 
(a)    Starting with row 1 of the matrix, we examine rows one by one until a row containing exactly
single zero elements are found. We make an experimental assignment (indicated by) to that cell.
Then we cross all other zeros in the column in which the assignment was made.
(b) When the set of rows has been completely examined an identical procedure is applied successively
to columns. Starting with Column 1, we examine columns until a column containing exactly one
remaining zero is found. We make an experimental assignment in that position and cross other
zeros in the row in which the assignment was made. It is found that no additional assignments are
possible. Thus, we have the complete �Zero assignment�,
A � I, B � III, C � II, D � IV
The minimum total man hours are computed as
Optimal assignment Man hours
A�I 8
B � III 4
jayaramulu123@gmail.com Page 41
Operation Research
C � II 19
D � IV 10
Total 41 hours
 
Example 2
A dairy plant has five milk tankers I, II, III, IV & V. These milk tankers are to be used on five delivery
routes A, B, C, D, and E. The distances (in kms) between dairy plant and the delivery routes are given in
the following distance matrix

  I II III IV V
A 160 130 175 190 200
B 135 120 130 160 175
C 140 110 155 170 185
D 50 50 80 80 110
E 55 35 70 80 105

 
How the milk tankers should be assigned to the chilling centers so as to minimize the distance travelled?
Solution
Step I: Subtracting minimum element in each row we get the first reduced matrix as

30 0 45 60 70
15 0 10 40 55
30 0 45 60 75
0 0 30 30 60
20 0 35 45 70
 
Step II: Subtracting minimum element in each column we get the second reduced matrix as
30 0 35 30 15
15 0 0 10 0
30 0 35 30 20
0 0 20 0 5
20 0 25 15 15

jayaramulu123@gmail.com Page 42
Operation Research
Step III: Row 1 has a single zero in column 2. We make an assignment by putting �   �around it and
delete other zeros in column 2 by marking �X�. Now column1 has a single zero in column 4 we make an
assignment by putting �   �and cross the other zero which is not yet crossed. Column 3 has a single zero
in row 2; we make an assignment and delete the other zero which is uncrossed. Now we see that there are
no remaining zeros; and row 3, row 5 and column 4 has no assignment. Therefore, we cannot get our
desired solution at this stage.
 
 
 

                                  
Step IV: Draw the minimum number of horizontal and vertical lines necessary to cover all zeros at least
once by using the following procedure
1.      Mark (√) row 3 and row 5 as having no assignments and column 2 as having zeros in rows 3 and 5.
2.      Next we mark (√) row 2 because this row contains assignment in marked column 2. No further
rows or columns will be required to mark during this procedure.
3.      Draw line L1 through marked col.2.
4.      Draw lines L2 & L3 through unmarked rows.
Step V: Select the smallest element say X among all uncovered elements which is X = 15. Subtract this
value X=15 from all of the values in the matrix not covered by lines and add X to all those values that lie at
the intersections of the lines L1, L2 & L3.
Applying these two rules, we get a new matrix
 
15 0 20 15 0
15 15 0 10 0
15 0 20 15 5
0 15 20 0 5
5 0 10 0 0

Step VI: Now reapply the test of Step III to obtain the desired solution.
jayaramulu123@gmail.com Page 43
Operation Research

The assignments are    

�������������� 

Total Distance 200 + 130 + 110 + 50 + 80 = 570

Module 7. Sequencing problem

Lesson 15
SOLUTION OF A SEQUENCING PROBLEM
15.1  Introduction
When a number of jobs are given to be done and they require processing on two or more machines, the
main concern of a manager is to find the order or sequence to perform these jobs. We shall consider the
sequencing problems in respect of the jobs to be performed in a factory and study the method of their
solution. Such sequencing problems can be broadly divided in two groups. In the first one, there are n jobs
to be done, each of which requires processing on some or all of the k different machines. We can determine
the effectiveness of each of the sequences that the technologically feasible (that is to say, those satisfying
the restrictions on the order in which each job must be processed through the machines) and choose a
sequence which optimizes the effectiveness. To illustrate, the timings of processing of each of the n jobs on
each of the k machines, in a certain given order, may be given and the time for performing the jobs may be
the measure of effectiveness. We shall select the sequences for which the total time taken in processing all
the jobs on the machines would be the minimum.
In this unit we will look into solution of a sequencing problem. In this lesson the solutions of following
cases will be discussed:
a)       n jobs and two machines A and B, all jobs processed in the order AB.
b)       n jobs and three machines A, B and C all jobs processed in the order ABC
c)       Problems with n jobs and m machines.
15.1.1  Processing of n jobs through two machines

jayaramulu123@gmail.com Page 44
Operation Research
The simplest possible sequencing problem is that of n job two machine sequencing problem in which we
want to determine the sequence in which n-job should be processed through two machines so as to
minimize the total elapsed time T. The problem can be described as:
a)   Only two machines A and B are involved;
b)   Each job is processed in the order AB.
c)   The exact or expected processing times A 1,A2,A3, --- , An ; B1,B2,B3, --- , Bn are known and are
provided in the following table
 
Machine Job(s)
1 2 3 -- - i -- - n
A A1 A2 A3 -- - Ai -- - An
B B1 B2 B3 -- - Bi -- - Bn
 
The problem is to find the sequence (or order) of jobs so as to minimize the total elapsed time T. The
solution of the above problem is also known as Johnson�s procedure which involves the following steps:
Step 1.      Select the smallest processing time occurring in the list A1,A2,A3, --- , An ; B1,B2,B3, --- , Bn if
there is a tie, either of the smallest processing times can be selected.
Step 2.       If the least processing time is A r , select the rth job first. If it is Bs, do the sth  job last  as the
given order is AB
Step 3.      There are now (n-1) jobs left to be ordered. Repeat steps I and II for the remaining set of
processing times obtained by deleting the processing time for both the machines
corresponding to the job already assigned.
Step 4.      Continue in the same manner till the entire jobs have been ordered. The resulting ordering will
minimize the total elapsed time T and is called the optimal sequence.
Step 5.      After finding the optimal sequence as stated above find the total elapsed time and idle times
on machines A and B as under:
Total The time between starting the first job in the optimal sequence on machine A and
elapsed completing the last job in the optimal machine B.
time =
Idle (Time when the last job in the optimal sequence on sequences is completed on
time on machine B)- (Time when the last job in the optimal sequences is completed on
machin machine A)
eA=
Idle (Time when the first job in the optimal sequences is completed on machine A)+
time on
machin
eB=   

jayaramulu123@gmail.com Page 45
Operation Research
The Johnson�s procedure can be illustrated by following examples:
Example 1
There are nine jobs, each of which must go through two machines P and Q in the order PQ, the processing
times (in hours) are given below:

Job(s)
Machine
A B C D E F G H I

P 2 5 4 9 6 8 7 5 4
Q 6 8 7 4 3 9 3 8 11

Find the sequence that minimizes the total elapsed time T. Also calculate the total idle time for the
machines in this period.
Solution
The minimum processing time on two machines is 2 which correspond to task A on machine P. This shows
that task A will be preceding first. After assigning task A, we are left with 8 tasks on two machines
Machine B C D E F G H I
P 5 4 9 6 8 7 5 4
Q 8 7 4 3 9 3 8 11
 
Minimum processing time in this reduced problem is 3 which correspond to jobs E and G (both on machine
Q). Now since the corresponding processing time of task E on machine P is less than the corresponding
processing time of task G on machine Q therefore task E will be processed in the last and task G next to
last. The situation will be dealt as
 
A             G E
 
The problem now reduces to following 6 tasks on two machines with processing time as follows:
Machine B C D F H I
P 5 4 9 8 5 4
Q 8 7 4 9 8 11
 
Here since the minimum processing time is 4 which occurs for tasks C and I on machine P and task D on
machine Q. Therefore, the task C which has less processing time on P will be processed first and then task
I and  task D will be placed at the last i.e., 7th sequence cell.
The sequence will appear as follows:
A C I       D E G
jayaramulu123@gmail.com Page 46
Operation Research
 
The problem now reduces to the following 3 tasks on two machines
 
Machine B F H
P 5 8 5
Q 8 9 8
 
In this reduced table the minimum processing time is 5 which occurs for tasks B and H both on machine P.
Now since the corresponding time of tasks B and H on machine Q are same i.e. 8. Tasks B or H may be
placed arbitrarily in the 4th and 5th sequence cells. The remaining task F can then be placed in the
6th sequence cell. Thus the optimal sequences are represented as 
 
A I C B H F D E G
or
A 1 C H B F D E G
Further, it is also possible to calculate the minimum elapsed time corresponding to the optimal sequencing
A → I → C → B → H → F → D → E → G.
Job Machine A Machine B
Sequence
Time In Time Out Time In Time Out
A 0 2 2 8
I 2 6 8 19
C 6 10 19 26
B 10 15 26 34
H 15 20 34 42
F 20 28 42 51
D 28 37 51 55
E 37 43 55 58
G 43 50 58 61
 
Hence the total elapsed time for this proposed sequence  staring from job A to completion of job G is 61
hours .During this time machine P remains idle for 11 hours (from 50 hours to 61 hours)and the machine Q
remains idle for 2 hours only (from 0 hour to 2 hour ).
15.2 Processing of n Jobs through Three Machines
The type of sequencing problem can be described as follows:
a)     Only three machines A, B and C are involved;
b)     Each job is processed in the prescribed order ABC

jayaramulu123@gmail.com Page 47
Operation Research
c)      No passing of jobs is permitted i.e. the same order over each machine is maintained.
d)     The exact or expected processing times A1,A2,A3, --- , An ; B1,B2,B3, --- , Bn and C1,C2,C3, --- ,
Cn are known and are denoted by the following table
 
Job(s)
Machine 1 2 3 -- - i - - n
-
A A1 A2 A3 -- - Ai -- - An
B B1 B2 B3 -- - Bi -- - Bn
C C1 C2 C3     Ci     Cn
 
Our objective will be to find the optimal sequence of jobs which minimizes the total elapsed time. No
general procedure is available so far for obtaining an optimal sequence in such case. However, the
Johnson�s procedure can be extended to cover the special cases where either one or both of the following
conditions hold:
 ���� a)      The minimum processing time on machine A ≥ the maximum processing time on machine
B.
b)      The minimum processing time on machine C ≥ the maximum processing time on machine B.
The method is to replace the problem by an equivalent problem involving n jobs and two machines. These
two fictitious machines are denoted by G and H and the corresponding time Gi and Hi are defined by
����������� Gi = Ai + Bi� and Bi + Ci
Now this problem with prescribed ordering GH is solved by the method with n jobs through two machines,
the resulting sequence will also be optimal for the original problem. The above methodology is illustrated
by following example:
Example 2
There are five jobs (namely 1,2,3,4 and 5), each of which must go through machines A, B and C in the
order ABC.  Processing Time (in hours) are given below:
Jobs 1 2 3 4 5
Machine A 5 7 6 9 5
Machine B 2 1 4 5 3
Machine C 3 7 5 6 7
 
Find the sequence that minimum the total elapsed time required to complete the jobs.
Solution

jayaramulu123@gmail.com Page 48
Operation Research
Here Min Ai = 5; Bi = 5 and Ci =3 since the condition of Min. A i ≥ Max. Bi is satisfied the given problem
can be converted into five jobs and two machines problem.
 
Job
s
1 7 5
2 8 8
3 10 9
4 14 11
5 8 10
 
The Optimal Sequence will be
2 5 4 3 1
Total elapsed Time will be
Machine A Machine B Machine C
Jobs
In Out In Out In Out

2 0 7 7 8 8 15
5 7 12 12 15 15 22
4 12 21 21 26 26 32
3 21 27 27 31 32 37
1 27 32 32 34 37 40
Min. total elapsed time is 40 hours.
Idle time for Machine A  is 8 hrs. (32-40)
Idle time for Machine B is 25 hours (0-7, 8-12, 15-21, 26-27, 31-32 and 34-40)
Idle time for Machine C is 12 hours (0-8, 22-26.)
15.3 Problems with n Jobs and m Machines
Let there be n jobs, each of which is to be processed through m machines, say M 1,M2, --- , Mm in the order
M1,M2,M3, --- , Mm. Let T ij be the time taken by the ith machine to complete the jth job.
The iterative procedure of obtaining an optimal sequence is as follows:
Step I: Find (i) minj (T1j)� ii) minj (Tmj)  iii) maxj (T2j,T3j,T4j, --- , T(m-1)j) for j=1,2,---, n
Step II: Check whether   
a.  minj(T1j)  ≥ maxj (Tij) for i=2,3,----,m-1
                                    Or 

jayaramulu123@gmail.com Page 49
Operation Research
b.   minj(Tmj)  ≥ maxj (Tij) for i=2,3,---,m-1
Step III: If the inequalities in Step II are not satisfied, method fails, otherwise, go to next step.
Step IV: Convert the m machine problem into two machine problem by introducing two fictitious
machines G and H, such that
����������������� TGj = T1j + T2j + --- +T(m-1)j� and� THj = T2j + T3j + --- +Tmj
Determine the optimal sequence of n jobs through 2 machines by using optimal sequence algorithm.
Step V: In addition to condition given in Step IV, if� Tij = T2j + T3j + --- +Tmj = C is a fixed positive
constant for all i = 1, 2, 3, � , n then determine the optimal sequence of n jobs and two
machines M1 and Mm in the order M1Mm by using the optimal sequence algorithm.
Example 3
Find an optimal sequence for the following sequencing problem of four jobs and five machines when
passing is not allowed, of which processing time (in hours) is given below:

Job Machine
A B C D E
1 7 5 2 3 9

2 6 6 4 5 10

3 5 4 5 6 8
4 8 3 3 2 6
 
Also find the total elapsed time.
Solution
Here Min. Ai = 5, Min. Ei = 6
Max. (Bi, Ci, Di) = 6, 5, 6�respectively
Since Min. Ei = Max. (Bi, Di) and Min. Ai = Max. Ci satisfied therefore the problem can be converted into 4
jobs and 2 fictitious machines G and H as follows:
   Fictitious Machine
Job

1 17 19
2 21 25
3 20 23
4 16 14
jayaramulu123@gmail.com Page 50
Operation Research
 
The above sequence will be:
1 3 2 4
Total Elapsed Time Corresponding to Optimal Sequence can be obtained as follows:
  Machine A   Machine B Machine C Machine D Machine E
Job In Out In Out In Out In Out In Out
1 0 7 7 12 12 14 14 17 17 26
3 7 12 12 16 16 21 21 27 27 35
2 12 18 18 24 24 28 28 33 35 45
4 18 26 26 29 29 32 33 35 45 51
 
Thus the minimum elapsed time is 51 hours.
Idle time for machine A = 25 hours(26-51)
Idle time for machine B = 33 hours(0-7,16-18,24-26,29-51)
Idle time for machine C = 37 hours(0-12,14-16,21-24,28-29,32-
51)
Idle time for machine D = 35 hours (0-14,17-21,27-28,35-51)
Idle time for machine E = 18 hours (0-17,26-27)

Module 6. Replacement theory

Lesson 13
REPLACEMENT OF ITEMS DETERIORATING WITH TIME
13.1 Introduction
In any establishment, sooner or later equipment needs to be replaced, particularly when new equipment
gives more efficient or economical service than the old one. In some cases, the old equipment might fail
and work no more or is worn out. In such situations it needs more expenditure on its maintenance than
before. The problem in such situation is to determine the best policy to be adopted with respect to
replacement of the equipment. The replacement theory provides answer to this question in terms of optimal
replacement period. Replacement theory deals with the analysis of materials and machines which
deteriorate with time and fix the optimal time of their replacement so that total cost is the minimum.
13.2 Replacement Decisions

jayaramulu123@gmail.com Page 51
Operation Research
The problem is to decide the best policy to adopt with regard to replacement. The need for replacement
arises in a number of different following situations so that different types of decisions may have to be
taken.
�         It may be necessary to decide whether to wait for a certain item to fail which might cause some
loss or to replace earlier at the expense of higher cost of the item.
�         The item can be considered individually to decide whether to replace now or if not when to
reconsider the item in question.
�         It is necessary to decide whether to replace by the same item or by a different type of item.
13.3 Types of Replacement Problems
i)        Replacement policy for items, efficiency of which declines gradually with time without change in
money value.
ii)      Replacement policy for items, efficiency of which declines gradually with time but with change in
money value.
iii)    Replacement policy of items breaking down suddenly
a)      Individual replacement policy
b)      Group replacement policy
iv)    Staff replacement
In this lesson we confine ourselves to first two situations only
13.4 Replacement of Items that Deteriorate with Time
There are certain items which deteriorate gradually with usage and such items decline in efficiency over a
period of time. Generally, the maintenance cost of certain items always increase gradually with time and a
stage comes when the maintenance cost becomes so large that it is better and economical to replace the
item with a new one. There may be number of alternatives and we may have a comparison between various
alternatives by considering the costs due to waste, scrap, loss of output, damage to equipment and safety
risks etc.
13.4.1 Replacement of items whose maintenance cost increases with time and the value of money
remains same during the period
The following costs are considered in such decisions:
C: Capital cost of a certain item say a machine
s(t): The selling or scrap value of the item after t years
f(t): Operating (or maintenance) cost of the item at time t
n: Optimal replacement period of the item
The operating cost function f(t) is assumed to be strictly positive. It may be continuous or discrete.
13.4.2 When t is a continuous variable
 Now the annual cost of the machine at time t is given by C � S(t) + f(t) and since the total maintenance
cost incurred on the machine during n years is  .     

jayaramulu123@gmail.com Page 52
Operation Research
Total cost T incurred on machine during n years is given by

��������������� 

Thus the average annual cost incurred on the machine per year during n years is given by

�����������   
To determine the value of optimal period (n), the principle of minima will be employed.

�����������   
��������������� 

�����������   
Clearly                                

�����������   
Therefore, it can be seen that A(n) or TA = f(n) is a minimum for T provided that f(t) is non�decreasing
and f(0) = 0. Hence, if time is measured continuously, then the average annual cost will be minimized by
replacing the item when the average cost becomes equal to the current maintenance cost.
13.4.3  When time is a discrete variable
Here the period of time is considered as fixed and n takes values 1,2,3, � then

�����������   
 By using finite differences, A(n) will be a minimum for that value of n for which

��������������� 

or          

����������� �
For this, we write

�����������   

�����������   

jayaramulu123@gmail.com Page 53
Operation Research

�����������   

�����������   

�����������   
Similarly, it can be shown

�����������   
This suggests the replacement policy that if time is measured in discrete units, then the average annual cost
will be minimized by replacing item when the next period�s maintenance cost become greater than the
current average cost. Hence, replace the equipment at the end of n years, if the maintenance cost in the
(n+1)th year is more than the average total cost in the (n)th year and the (n)th year�s maintenance cost is less
than the previous year�s average total cost. The following examples will illustrate this methodology
Example 1  A milk plant is considering replacement of a machine whose cost price is Rs. 12,200 and the
scrap value Rs. 200. The running (maintenance and operating) costs in Rs. are found from experience to be
as follows:
Year: 1 2 3 4 5 6 7 8
Running Cost: 200 500 800 1200 1800 2500 3200 4000
When should the machine be replaced?
Solution The computations can be summarized in the following tabular form:
Table 13.1 Calculations for average cost of machine
  (In Rupees)

Year  Running Cumulative Depreciation Cost Total Cost TC Average


 
Cost Running (5) = (3) + (4) Cost
(1) Cost  (4)
(2) (3) (6) = (5)/(1)
1 200 200 12000 12200 12200
2 500 700 12000 12700 6350
3 800 1500 12000 13500 4500
4 1200 2700 12000 14700 3675
5 1800 4500 12000 16500 3300
6 2500 7000 12000 19000 3167
7 3200 10200 12000 22200 3171
8 4000 14200 12000 26200 3275
 

jayaramulu123@gmail.com Page 54
Operation Research
From the table it is noted that the average total cost per year, A(n) is minimum in the 6 th year (Rs. 3167).
Also the average cost in 7th year (Rs.3171) is more than the cost in 6 th year. Hence the machine should be
replaced after every 6 years.
Example 2
A Machine owner finds from his past records that the maintenance costs per year of a machine whose
purchase price is Rs. 8000 are as given below:
Year: 1 2 3 4 5 6 7 8
Maintenance Cost: 1000 1300 1700 2200 2900 3800 4800 6000
Resale Price: 4000 2000 1200 600 500 400 400 400
Determine at which time it is profitable to replace the machine.
Solution C = Rs. 8000. Table 13.2 shows  the average cost per year during the life of machine. Here, The
computations can be summarized in the following tabular form:
Table 13.2 Calculations for average cost of machine

Year  f(t) Cumulative Scrap Total cost


maintenance value 
cost
1 1000 1000 4000 5000 5000
2 1300 2300 2000 8300 4150
3 1700 4000 1200 10800 3600
4 2200 6200 600 13600 3400
5 2900 9100 500 16600
6 3800 12900 400 20500 3417
7 4800 17700 400 25300 3614
8 6000 23700 400 31300 3913
 
The above table shows that the value of T A during fifth year is minimum. Hence the machine should be
replaced after every fifth year.
Example 3
The cost of a machine is Rs. 6100 and its scrap value is only Rs.100. The maintenance costs are found to
be
Year: 1 2 3 4 5 6 7 8
Maintenance Cost (in Rs.): 100 25 400 600 900 125 1600 2000
0 0
When should the Machine be replaced?
Solution

jayaramulu123@gmail.com Page 55
Operation Research
  C = 6100, s(t) = 100 The computations can be summarized in the following tabular form:
Table 13.3 Calculations for average cost of machine
Replace Cumulative Total cost
at the end maintenance cost
of year

1 100 100 6100 6100


2 250 350 6350 3175
3 400 750 6750 2250
4 600 1350 7350 1737.50
5 900 2250 8250 1650
6 1250 3500 9500
7 1600 5100 11100 1585.7
8 2000 7100 13100 1637.50
 
It is now observed that the machine should be replaced at the end of sixth year otherwise the average cost
per year will start to increase.
13.4.4 Replacement of items whose maintenance cost increases with time and the money value
changes at a constant rate
To understand this let us define the following terms:
Money Value
Since money has a value over time, therefore the explanation of the statement: �Money is worth 10% per
year� can be given in two ways:
(a)   In one way, spending Rs.100 today would be equivalent to spend Rs.110 in year�s time. In other
words if we plan to spend Rs.110 after a year from now, we could spend Rs.100 today and an
investment which would be worth Rs.110 next year.
(b)  Alternatively if we borrow Rs.100 at the interest of 10% per year and spend Rs.100 today, we have
to pay Rs.100 after one year (next year).
Thus, we conclude that Rs.100 is equal to Rs.110 a year from now. Consequently Rs. 1 from a year now is
equal to (1+0.1)-1 rupee today.
Present Worth Factor
As we have seen, a rupee a year from now will be equivalent to (1+0.1) -1 rupee today at the discount rate of
10% per year. So, one rupee in n years from now will be equal to (1+0.1)-n. Therefore, the quantity (1+0.1)-
n
 is called the Present Worth Factor (PWF) or Present Value (PV) of one rupee spent in n years from now.
In general, if r is the rate of interest, then (1+r)-n is called PWF or PV of one rupee spent in n years from
now onwards. The expression (1+r)-n is known as compound amount factor of one rupee spent in n years.
jayaramulu123@gmail.com Page 56
Operation Research
Discount Rate
Let r be the rate of interest. Therefore present worth factor of unit amount to be spent after one year

is  .
Then v is known as the discount rate. The optimum replacement policy for replacement of item where
maintenance costs increase with time and money value changes with constant rate can be determined by
following method:
Suppose that the item (which may be machine or equipment) is available for use over a series of time
periods of equal intervals (say one year). Let
C = Purchase price of the item to be replaceds
Rt = Running (or maintenance) cost in the tth year
R = Rate of interest  

�is the present worth of a rupee to be spent in a year hence.


Let the item be replaced at the end of every nth year. The year wise present worth of expenditure on the
item in the successive cycles of n years can be calculated as follows:
 

Year 1 2---- n n+1 n+2 ---- 2n 2n+1

Present C+R1 R2v Rnvn-1 (C+R1)vn R2vn+1   Rnv2n-1 (C+R1)v2n


worth

 
Assuming that machines has no resale value at the time of replacement, the present worth of the machine
in n years will be given by
��������������� 

Summing up the right-hand side, column-wise


����������� 

��������������� 

�����������  ,
using sum of an infinite G.P.

jayaramulu123@gmail.com Page 57
Operation Research

�����������   

�����������   
f(n) and f(n+1) given above at n = 0,1,2�, are called the weighted average cost of previous n years with
weights 1,v, v2, ----vn-1respectively. P(n) is the amount of money required now to pay all the future costs of
acquiring and operating the equipment when it is renewed every n years. However, if P (n) is less than P
(n+1) then replacing the equipment each n year is preferable to replacing each n years is preferable to
replacing each (n+1) years. Further, if the best policy is replacing every n years, then the two inequalities P
(n+1) � P (n) > 0 and P (n-1) - P (n) < 0 must hold, without giving the proof we shall state the following
two inequalities which holds good at n, the optimal replacement interval.

�����������   

�����������   
As a result of these two inequalities, rules for minimizing costs may be stated as follows:

1.      Do not replace if the operating cost of next period is less than the weighted average of previous
costs.

2.      Replace if the operating cost of the next period is greater than the weighted average of the previous
costs.

Working Procedure
The step-by-step procedure for solving the problem is stated as under:
1. Write in a column the running/maintenance costs of machine or equipment for different years,
Rn.

2. In the next column write the discount factor indicating the present value of a rupee received after
(i-1) years,  

3. The two column values are multiplied to get present value of the maintenance costs, i.e., .
th
4. These discounted maintenance costs are then cumulated to the i  year to get .

5. The cost of machine or equipment is added to the values obtained in Step 4 above to  

Obtain C+  .

6. The discount factors are then cumulated to get  .

jayaramulu123@gmail.com Page 58
Operation Research
7. The total costs obtained in (Step 5) are divided by the corresponding value of the accumulated
discount factor for each of the years.

8. Now compare the column of maintenance costs which is constantly increasing with the last
column. Replace the machine in the latest year that the last column exceeds the column of
maintenance costs.

Example 4

A milk plant is offered an equipment A which is priced at Rs.60,000 and the  costs of operation and maintenance
are estimated to be Rs.10,000 for each of the first 5 years, increasing every year by Rs. 3000 per year in the
sixth and subsequent years. If money carries the rate of interest 10% per annum what would the optimal
replacement period?
Solution
Table 13.4 Determination of optimal replacement period
At the end Operating & Discounte Discounted Cumulative Discounted total Cumulative Weighted
of year maintenance d factor operation & Discounted cost discounted average
(n) cost   maintenance operation & factor annual cost
Rn maintenanc
cost  
e cost

(1) (2) (3) (4)=(2)x(3) (5) (6)=(5)+60000 (7) (8)=(6)+(7)


1 10000 1.0000 10000.00 10000.00 70000.00 1.00 70000.00
2 10000 0.9091 9091.00 19091.00 79091.00 1.91 41428.42
3 10000 0.8264 8264.00 27355.00 87355.00 2.74 31933.83
4 10000 0.7513 7513.00 34868.00 94868.00 3.49 27207.75
5 10000 0.6830 6830.00 41698.00 101698.00 4.17 24389.18
6 13000 0.6209 8071.70 49769.70 109769.70 4.79 22913.08
7 16000 0.5645 9032.00 58801.70 118801.70 5.36 22184.36
8 19000 0.5132 9750.80 68552.50 128552.50 5.87 21905.89
9 22000 0.4665 10263.00 78815.50 138815.50 6.33 21912.82
10 25000 0.4241 10602.50 89418.00 149418.00 6.76 22106.52
 
From Table 13.4 we find the weighted cost is minimum at the end of 8 th year, hence the equipment should
be replaced at the end of 8th year.
Example 5
A Manufacturer is offered two machines A and B. Machine A is priced at Rs. 5000 and running cost is
estimated at Rs. 800 for each of the first five years, increasing by Rs. 200 per year in the sixth and
subsequent years. Machine B, with the same capacity as A, costs Rs. 2500, but has running cost of Rs.
1200 per year for six years, thereafter increasing by Rs. 200 per year. If money is worth 10% per year,

jayaramulu123@gmail.com Page 59
Operation Research
which machine should be purchased? (Assume that the machines will eventually be sold for scrap at a
negligible price).
Solution

Since money is worth 10% per year, therefore discount rate is 
Table 13.5 Computation of weighted average cost for machine A
At the Operating & Discounted Discounted Cumulative Discounted Cumulativ Weighted
end of maintenance factor operation & Discounted total cost e average
year cost   maintenance discounted annual cost
operation &
(n) Rn factor
cost   maintenance
cost
(1) (2) (3) (4)=(2)x(3) (5) (6)=(5)+ (7) (8)=(6)+(7)
       5000
1 800 1.0000 800 800 5800 1 5800
2 800 0.9091 727 1527 6527 1.9091 3419.035
3 800 0.8264 661 2188 7188 2.7355 2627.819
4 800 0.7513 601 2789 7789 3.4868 2233.98
5 800 0.6830 546 3336 8336 4.1698 1999.098
6 1000 0.6209 621 3957 8957 4.7907 1869.61
7 1200 0.5645 677 4634 9634 5.3552 1799.025
8 1400 0.5132 718 5353 10353 5.8684 1764.13
9 1600 0.4665 746 6099 11099 6.3349 1752.043
10 1800 0.4241 763 6862 11862 6.759 1755.053
 
From table 13.5 we conclude that for machine A 1600<1752.043<1800. Since the running cost of 9 th year
is 1600and that of 10th year is 1800 and 1800>1752.043, it would be economical to replace machine A at
the end of nine years.
Table 13.6 Computation of weighted average cost for machine B
At the Operating & Discounted Discounted Cumulative Discounted total Cumulative Weighted
end of maintenance factor operation & Discounted cost discounted average
year cost   maintenance operation & factor annual cost
(n) Rn maintenance
cost  
cost

(1) (2) (3) (4)=(2)x(3) (5) (6)=(5)+ (7) (8)=(6)+(7)


       2500
1 1200 1.0000 1200.00 1200.00 3700.00 1.00 3700.00
2 1200 0.9091 1090.92 2290.92 4790.92 1.91 2509.52
3 1200 0.8264 991.68 3282.60 5782.60 2.74 2113.91
4 1200 0.7513 901.56 4184.16 6684.16 3.49 1916.99
5 1200 0.6830 819.60 5003.76 7503.76 4.17 1799.55
jayaramulu123@gmail.com Page 60
Operation Research
6 1200 0.6209 745.08 5748.84 8248.84 4.79 1721.84
7 1400 0.5645 790.30 6539.14 9039.14 5.36 1687.92
8 1600 0.5132 821.12 7360.26 9860.26 5.87 1680.23
9 1800 0.4665 839.70 8199.96 10699.96 6.33 1689.05
10 2000 0.4241 848.20 9048.16 11548.16 6.76 1708.56
 
In table13.6 we find that 1800<1689<2300 so it is better to replace the machine B after 8 th year. The
equivalent yearly average discounted value of future costs is Rs. 1748.60 for machine A and it is
1680.23for machine B. Hence, it is more economical to buy machine B rather than machine A.

GameTheory
1. Introduction to Game Theory
Game theory is a type of decision theory in which one’s choice of action is determined after taking
into account all possible alternatives available to an opponent playing the same game, rather than just
by the possibilities of several outcome results. Game theory does not insist on how a game should be
played but tells the procedure and principles by which action should be selected. Thus it is a decision
theory useful in competitive situations.

Game is defined as an activity between two or more persons according to a set of rules at the end of
which each person receives some benefit or suffers loss. The set of rules defines the game. Going
through the set of rules once by the participants defines a play.

2. Properties of a Game
a) There are finite numbers of competitors called ‘players’
b) Each player has a finite number of possible courses of action called ‘strategies’
c) All the strategies and their effects are known to the players but player does not know which
strategy is to be chosen.
d) A game is played when each player chooses one of his strategies. The strategies are assumed
jayaramulu123@gmail.com Page 61
Operation Research
to be made simultaneously with an outcome such that no player knows his opponents strategy
until he decides his own strategy.
e) The game is a combination of the strategies and in certain units which determines the gain or
loss.
f) The figures shown as the outcomes of strategies in a matrix form are called ‘pay-off matrix’.
g) The player playing the game always tries to choose the best course of action which results in
optimal pay off called ‘optimal strategy’.
h) The expected pay off when all the players of the game follow their optimal strategies is
known as ‘value of the game’. The main objective of a problem of a game is to find the value
of the game.

jayaramulu123@gmail.com Page 62
i) The game is said to be ‘fair’ game if the value of the game is zero otherwise it s known as
‘unfair’.
3. Characteristics of Game Theory
a) Competitive game: A competitive situation is called a competitive game if it has the following
four properties
 There are finite number of competitors such that n ≥ 2. In case n = 2, it is called a two-
person game and in case n > 2, it is referred as n-person game.
 Each player has a list of finite number of possible activities.
 A play is said to occur when each player chooses one of his activities. The choices are
assumed to be made simultaneously i.e. no player knows the choice of the other until he
has decided on his own.
 Every combination of activities determines an outcome which results in a gain of
payments to each player, provided each player is playing uncompromisingly to get as
much as possible. Negative gain implies the loss of same amount.
b) Strategy: The strategy of a player is the predetermined rule by which player decides his course
of action from his own list during the game.
The two types of strategy are
 Pure strategy
 Mixed strategy
Pure Strategy: If a player knows exactly what the other player is going to do, a deterministic
situation is obtained and objective function is to maximize the gain. Therefore, the pure
strategy is a decision rule always to select a particular course of action.
Mixed Strategy: If a player is guessing as to which activity is to be selected by the other on
any particular occasion, a probabilistic situation is obtained and objective function is to
maximize the expected gain. Thus the mixed strategy is a selection among pure strategies with
fixed probabilities.
c) Number of persons: A game is called ‘n’ person game if the number of persons playing is ‘n’.
The person means an individual or a group aiming at a particular objective.
Two-person, zero-sum game: A game with only two players (player A and player B) is called
a ‘two-person, zero-sum game’, if the losses of one player are equivalent to the gains of the
other so that the sum of their net gains is zero.
Two-person, zero-sum games are also called rectangular games as these are usually
represented by a payoff matrix in a rectangular form.
d) Number of activities: The activities may be finite or infinite.
e) Payoff: The quantitative measure of satisfaction a person gets at the end of each play is called
a payoff
f) Payoff matrix: Suppose the player A has ‘m’ activities and the player B has ‘n’ activities.
Then a payoff matrix can be formed by adopting the following rules
 Row designations for each matrix are the activities available to player A
 Column designations for each matrix are the activities available to player B
 Cell entry Vij is the payment to player A in A’s payoff matrix when A chooses the
activity i and B chooses the activity j.
 With a zero-sum, two-person game, the cell entry in the player B’s payoff matrix will be
negative of the corresponding cell entry Vij in the player A’s payoff matrix so that sum of
payoff matrices for player A and player B is ultimately zero.
g) Value of the game: Value of the game is the maximum guaranteed game to player A
(maximizing player) if both the players uses their best strategies. It is generally denoted by
‘V’ and it is unique.

4. Classification of Games
All games are classified into
 Pure strategy games
 Mixed strategy games
Strategy: It is the pre-determined rule by which each player decides his course of action from his list
available to him. How one course of action is selected out of various courses available to him is
known as strategy of the game.
Types of Strategy: Generally two types of strategy are employed
(i) Pure Strategy
(ii) Mixed Strategy
(i) Pure Strategy: It is the predetermined course of action to be employed by the player. The
players knew it in advance. It is usually represented by a number with which the course of
action is associated.
(ii) Mixed Strategy: In mixed strategy the player decides his course of action in accordance
with some fixed probability distribution. Probability are associated with each course of
action and the selection is done as per these probabilities.
In mixed strategy the opponent cannot be sure of the course of action to be taken on any particular
occasion. Pure strategy games can be solved by saddle point method.
Decision of a Game. In Game theory, best strategy for each player is determined on the basis of some
rule. Since both the players are expected to be rational in their approach this is known as the criteria
of optimality. Each player lists the possible outcomes from his action and selects the best action to
achieve his objectives. This criteria of optimality is expressed as Maximin for the maximising player
and Minimax for the minimising player.

5. The Maximin-Minimax Principle


a) Maximin Criteria: The maximising player lists his minimum gains from each strategy and
selects the strategy which gives the maximum out of these minimum gains.
b) Minimax Criteria : The minimising player lists his maximum loss from each strategy and
selects the strategy which gives him the minimum loss out of these maximum losses.
For Example Consider a two person zero sum game involving the set of pure strategy for Maximising
player A say Al A2 & As and for player B, B1 & B2, with the following payoff
Player B
B1 B2 Row MaxMin
A1 9 2 2
Player A A2 8 6 6* Maximin
A3 6 4 4
Column MiniMax 9 6* Minimax

Since Maximin = Minimax V = 6

Suppose that player A starts the game knowing fully well that whatever strategy he adopts B will
select that particular counter strategy which will minimise the payoff to A. If A selects the strategy Al
then B will select B2 so that A may get minimum gain. Similarly if A chooses A2 than B will adopt
the strategy of B2. Naturally A would like to maximise his maximin gain which is just the largest of
row minima. Which is called 'maximin strategy'. Similarly B will minimise his minimum loss which
is called 'minimax strategy'. We observe that in the above example, the maximum if row minima and
minimum of column maxima are equal. In symbols.
Maxi [Min.] = Mini [Max]
The strategies followed by both the p1ayersarecalled ‘optimum strategy’.
Value of Game. In game theory, the concept value of game is considered as very important. The
value of game is the maximum guaranted gain to the maximising player if both the players use their
best strategy. It refers to the average payoff per play of the game over a period of time. Consider the
following the games.
Player Y Player Y
3 4 -7 2
Player X -6 -2 Player X -3 -1

(With Positive Value) (With Negative Value)

In the first game player X wins 3 points and the value of the value is three with positive sign and in
the second game the player Y wins 3 points and the value of the game is -ve which indicates that Y is
the Winner. The value is denoted by 'v.
The different methods for solving a mixed strategy game are
• Analytical method
• Graphical method
• Dominance rule
6. Two-Person and Zero-Sum Game

Two-person zero-sum games may be deterministic or probabilistic. The deterministic games will
have saddle points and pure strategies exist in such games. In contrast, the probabilistic games will
have no saddle points and mixed strategies are taken with the help of probabilities.
Definition of saddle point

A saddle point of a matrix is the position of such an element in the payoff matrix, which is
minimum in its row and the maximum in its column.
Procedure to find the saddle point

 Select the minimum element of each row of the payoff matrix and mark them
with circles.
 Select the maximum element of each column of the payoff matrix and mark them with
squares.
 If their appears an element in the payoff matrix with a circle and a square together
then that position is called saddle point and the element is the value of the game.
Solution of games with saddle point

To obtain a solution of a game with a saddle point, it is feasible to find out

 Best strategy for player A


 Best strategy for player B
 The value of the game
The best strategies for player A and B will be those which correspond to the row and column
respectively through the saddle point.

Example 1: Solve the payoff matrix

Player B

I II III IV V
I
II
Player A III
IV

Solution:
-2 0 0 5 3
3 2 1 2 2
-4 -3 0 -2 6
5 3 -4 2 -6
Player B
I II III IV V Row MaxMin
I -2 0 0 5 3 -2
Player A II 3 2 1 2 2 1 Maxmin Value

III -4 -3 0 -2 6 -4

IV 5 3 -4 2 -6 -6
Column MiniMax 5 3 1 5 6

Minimax value

Strategy of player A – II
Strategy of player B - III
Value of the game = 1

Example -2: Solve the payoff matrix

B1 B2 B3 B4

A1 1 7 3 4

A2 5 6 4 5

A3 7 2 0 3

Solution

B1 B2 B3 B4 Row MaxMin

A1 1 7 3 4 1
A2 4 4 Maximin Value
5 6 5

A3 7 2 0 3 0
Column MiniMax 7 7 4 5
Minimax Value
Strategy of player A= A2
Strategy of player B = B3
Value of the game = 4

Points to remember

(i) Saddle point mayor may not exist in a given game.

(ii) There may be more than one saddle point then there will be more than one solution. (Such
situation is rare in the rea1life).
(iii) The value of game may be +ve or -ve.
(iv) The value of game may be zero which means 'fair game'.

II GAMES WITH MIXED STRATEGIES

All game problems, where saddle point does not exist are taken as mixed strategy problems. Where
row minima is not equal to column maxima, then different methods are used to solve the different
types of problems. Both players will use different strategies with certain probabilities to optimise. For
the solution of games with mixed strategies, any of the following methods can be applied.

1. ODDS METHOD (2x2 game without saddle point)


2. Dominance Method.
3. Sub Games Method. – For (mx2) or (2xn) Matrices
4. Equal Gains Method.
5. Linear Programming Method-Graphic solution
These methods are explained one by one with examples, in detail.

1. ODDS Method - For 2 x 2 Game


Use of odds method is possible only in case of games with 2 x 2 matrix. Here it should be ensured
that the sum of column odds and row odds is equal.

METHOD OF FINDING OUT ODDS


Step 1: Find out the difference in the value of in cell (1, 1) and the value in the cell (1,2) of the first
row and place it in front of second row.
Step 2: Find out the difference in the value of cell (2, 1) and (2, 2) of the second row and place it in
front of first row.
Step 3: Find out the differences in the value of cell (1, 1) and (2, 1) of the first column and place it
below the second column.
Step 4: Similarly find the difference between the value of the cell (1, 2) and the value in cell (2, 2) of
the second column and place it below the first column.
The above odds or differences are taken as positive (ignoring the negative sign)

Strategy  Y
 Y1 Y2 ODDS
X X1 a1 a2  (b1 – b2)
X2 b1 b2  (a1 – a2)
 
ODDS (a2 – b2) (a1 – b1)

The value of game is determined with the help of following equation.


2. Dominance Method. Dominance method is also applicable to pure strategy and mixed strategy
problem. In pure strategy the solution is obtained by itself while in mixed strategy it can be used for
simplifying the problem.

Principle of Dominance. The Principle of Dominance states that if the strategy of a player dominates
over the other strategy in all condition then the later strategy is ignored because it will not effect the
solution in any way. For the gainer point of view if a strategy gives more gain than another strategy,
then first strategy dominates over the other and the second strategy can be ignored altogether.
Similarly from loser point of view, if a strategy involves lesser loss than other in all condition then
second can be ignored. So determination of superior or inferior strategy is based upon the objective of
the player. Since each player is to select his best strategy, the inferior strategies can be eliminated. In
other words, ineffective rows & column can be deleted from the game matrix and only effective rows
& columns of the matrix are retained in the reduced matrix.

For deleting the ineffective rows & columns the following general rules are to be followed.
1) If all the elements of a row (say i th row) of a pay off matrix are less than or equal to ( ) the
corresponding each element of the other row (say jth row) then the player A will never choose
the ith strategy OR ith row is dominated by jth row. Then delete ith row.

2) If all the elements of a column (say j th column are greater than or equal to the corresponding
elements of any other column (say jth column) then ith column is dominated by jth column.
Then delete ith column.

3) A pure strategy of a player may also be dominated if it is inferior to some convex combination
of two or more pure strategies. As a particular case, if all the elements of a column are greater
than or equal to the average of two or more other columns then this column is dominated by
the group of columns. Similarly if all the elements of row are less than or equal to the average
of two or more rows then this row is dominated by other group of row.
4) By eliminating some of the dominated rows a columns and if the game is reduced to 2 x 2
form it can be easily solved by odds method.
Since there is no saddle point, so we apply dominance method. Here Row II dominates Row II so we
will delete Row II.
B
I II III
I
A 5 20 -10
III 20 15 18

Since column III dominates column I, we delete column I we get:


B
II III odds
I 20 -10 3
A
III 15 18 30
Odds 28 5 33
Using the dominance property obtain the optimal strategies for both the players and determine the
value of the game. The pay off matrix for player A is given.

Example 2:
Operation Research

Graphical method
The graphical method is used to solve the games whose payoff matrix has
 Two rows and n columns (2 x n)
 m rows and two columns (m x 2)
Algorithm for solving 2 x n matrix games

 Draw two vertical axes 1 unit apart. The two lines are x1 = 0, x1 = 1
 Take the points of the first row in the payoff matrix on the vertical line x1 = 1 and the
points of the second row in the payoff matrix on the vertical line x1 = 0.
 The point a1j on axis x1 = 1 is then joined to the point a2j on the axis x1 = 0 to give a
straight line. Draw ‘n’ straight lines for j=1, 2… n and determine the highest point of the
lower envelope obtained. This will be the maximin point.
 The two or more lines passing through the maximin point determines the required 2 x 2
payoff matrix. This in turn gives the optimum solution by making use of analytical
method.
Example 1
Solve by graphical method

Solution:

Jayaramulu123@gmail.com Page 15
V = 66/13
SA = (4/13, 9 /13)
SB = (0, 10/13, 3 /13)

Example 2

Solve by graphical method

Solution:
V = 8/7
SA = (3/7, 4/7)
SB = (2/7, 0, 5 /7)

Algorithm for solving m x 2 matrix games


 Draw two vertical axes 1 unit apart. The two lines are x1 =0, x1 = 1
 Take the points of the first row in the payoff matrix on the vertical line x1 = 1 and the
points of the second row in the payoff matrix on the vertical line x1 = 0.
 The point a1j on axis x1 = 1 is then joined to the point a2j on the axis x1 = 0 to give a
straight line. Draw ‘n’ straight lines for j=1, 2… n and determine the lowest point of the
upper envelope obtained. This will be the minimax point.
 The two or more lines passing through the minimax point determines the required 2 x 2
payoff matrix. This in turn gives the optimum solution by making use of analytical
method.

Example 1
Solve by graphical method
Solution

V = 3/9 = 1/3
SA = (0, 5 /9,
4/9, 0) SB =
(3/9, 6 /9)

Example 2: Solve by graphical method


Operation Research

Solution:

V = 73/17
SA = (0,
16/17, 1/17,
0, 0) SB =
(5/17, 12 /17)

Jayaramulu123@gmail.com Page 19
Operation Research

Module 3. Inventory control

Lesson 11
ECONOMIC LOT SIZE MODELS WITH KNOWN DEMAND
11.1 Introduction
In the last lesson we have seen that maintenance of proper Inventory Control System helps in
keeping the investment in Inventories as low as possible and yet (i) ensures availability of
materials by providing adequate protection against supply uncertainty and consumption of
materials and (ii) allows full advantages of economies of bulk purchases and transportation
costs. The basic Inventory Control problem therefore lies in determining firstly when should
an order for materials be placed and secondly how much should be produced at the beginning
of each time interval or what quantity of an item should be ordered each time. In this lesson
we will learn how to develop inventory model.
11.2 Variables in Inventory Problem
The variables used in any inventory model are of two types: Controlled and Uncontrolled
variables
11.2.1 Controlled variables
The following are the variables that may be considered separately or in combination:
�         How much quantity acquired
�         The frequency or timing of acquisition .How often or when to replenish the
inventory?
�         The completion stage of stocked items.
11.2.2 Uncontrolled variables
The following are the principal variables that may be controlled:
�         The holding costs, shortage or penalty cost, set up costs.
�         Demand: It is the number of units required per period and may be either known
exactly or is known in terms of probabilities or is completely unknown. Further if the
demand is known, it may be either fixed or variable per unit of time. The model which
has fixed demand is known as deterministic model.
�      Lead Time: This is the time of placing an order and its arrival in stock as shown in
Fig. 11.1. If the lead time is known and not equal to zero and if demand is
deterministic then one should order in advance by an amount of time equal to lead
time. If the lead time is zero then there is no need to order in advance. If lead time is
variable then it is known as probabilistically.

Jayaramulu123@gmail.com Page 20
Operation Research

Fig. 11.1 Inventory with constant demand rate and constant lead time
�         Amount delivered (supply of goods) : The supply of goods may be instantaneous
or spread over a period of time .If a quantity q is ordered for purchase, the amount
delivered may very around g with known probability density function .
11.3 Symbols Used in Inventory Models
C1 = Holding cost per item per unit time
C2 = Shortage cost per unit quantity per unit time for the back-log case and� per unit
only for no back log case.
C3 = Set up cost per production run
R = Demand Rate
K = Production Rate
t = Scheduling time period which is not prescribed.
tp = Scheduling time period (prescribed)
z = Order Level (or stock level)
D = Total Demand
Q = Quantity already present in the beginning
L = Lead time
11.4 Classification of Inventory Models
The inventory models are broadly classified as either deterministic (variables are known with
certainty) or stochastic (variables are probabilistic). The deterministic models are further
divided into Static demand models and dynamic demand models. In this lesson we will
discuss deterministic inventory model i.e. economic lot size system with uniform demand.
11.5 Economic Order Quantity (EOQ) Model
This concept was developed by F. Haris in 1916. The concept is as lot size (q) increases the
carrying charges (C1) will increase while the ordering cost (C3) will decrease. On the other
hand as the lot size (q) decreases, the carrying cost (C 1) will decrease but the ordering costs
will increase. Economic Ordering Quantity (EOQ) is the size of order which minimizes total
Jayaramulu123@gmail.com Page 21
Operation Research

annual cost of carrying inventory and cost of ordering under the assumed conditions of
certainly and annual demands are known.

11.5.1 The economic lot size system with uniform demand


In this model we want to derive an Economic Lot Size Formula for the optimum production
quantity q per cycle (per production run) of a single product so as to minimize the total
average variable cost per unit time. The model is based on the following basic assumptions:
i)     The demand for the item is uniform at the rate of R quantity units per unit time
ii)    The lead time is known and fixed. Thus, when the lead time is zero, the delivery of
item is instantaneous.
iii)   Production rate is infinite i.e. production is instantaneous
iv)   Shortages are not allowed
v)     Holding cost is Rs. C1 in per quantity unit per unit time
vi)   Set up cost is Rs. C3 per set up.
This can be solved by two following methods
11.5.1.1 Algebraic method
The algebraic method is based on the following principle:
Inventory carrying costs=Annual ordering (set up) costs                                       (Eq.
11.1)
Since the demand is uniform and known exactly and supply is instantaneous, the reorder
point is that when inventory falls to zero.
Average Inventory =1/2(maximum level +minimum level) = (q+0)/2=q/2
Total inventory carrying costs are determined by using the following formula:
�� (Total inventory carrying costs per unit) = (average no. of units in inventory) x (costs of
one unit)
�� �(inventory carrying costs percentage)
=1/2q.C.I=1/2qC1                                                     ������������������������������������� ��
������������������������������������������������������������������

���������������������� ���������������  ��� (Eq. 11.2)


Where C1 = CI is holding or carrying cost per unit for unit time.
Total annual ordering costs are obtained as follows:
����������� Total annual ordering costs = (number of orders per
year) � (ordering cost per order) = (R/q) . C3 = (R/q)
C3����� ��������������������������������
��������������������������� ����������� 
��������(Eq. 11.3)
Jayaramulu123@gmail.com Page 22
Now summing up the total inventory carrying cost and total ordering cost we get total
inventory cost as   
Operation Research

�����������   Total ordering costs = (Total inventory carrying costs per unit) +


(Total annual ordering costs)
��������������� ���� C(q)= 1/2qC1+(R/q)C3  , this is cost
equation .    ��(Eq. 11.4)
The total inventory cost C(q) is minimum when the  inventory carrying costs become equal to
the total ordering costs. Therefore

1/2qC1=(R/q)C3  or  ��������������� ����(Eq. 11.5)

  Or optimal 
To find the minimum of total inventory cost C(q) , we substitute the value of q  from (11.5) in
cost equation (11.4) we get

�����������  �                           ��� (Eq. 11.6)

Optimum inventory cost (Cmin) =  


To obtain the optimum interval of ordering (t*) we have
(Economic ordering quantity) = (demand rate) � (interval of ordering)
��������������� q=R.t                                    � (Eq. 11.7)

  ����
(Eq. 11.8)
11.5.1.2 Calculus method
Let each production cycle be made at fixed interval�t� and therefore the quantity q already
present in the beginning (when the business was started) should be
����������� Q= R.t                                                         ��� (Eq. 11.9)
where R is the demand rate.
Since the stock in small time dt will be Rt.dt, therefore the stock in total time t will

be  �=Area of the inventory ∆POA as shown in Fig. 11.2

Jayaramulu123@gmail.com Page 23
Operation Research

Fig. 11.2 Inventory profile of EOQ model


q = Rt

The rate of replenishment = slope of line 


Thus the cost of holiday inventory is = c1(Area of ∆OPA) per production run

�����������                                                          �


������� ���� (Eq. 11.10)
Set up cost is C3 per production run for interval t.                           
���������������������������� ����� (Eq. 11.11)
Total cost C(t) is summing up the costs in (11.10) and (11.11) and dividing by t we get the
average total cost given by

�����������                                          
������������� ������(Eq. 11.12)
The condition for minimum or maximum of C(t) is

��������������� 

���������������  ��� (Eq. 11.13)

����������� 
����������� 

Jayaramulu123@gmail.com Page 24
����������                                               
����� ����������� �����������(Eq. 11.14)
Operation Research

������������� 

��
���� ��� (Eq. 11.15)
which is known as optimal lot size formula.Therefore

��������������� 

per unit time is obtained by putting in equation (11.12).The above procedure is illustrated
through following examples
Example 1 A manufacture has to supply his customer with 600 units of a product per year,
shortages are not allowed and the storage costs amounts to 0.60 per unit per year. The set up
cost per run is Rs. 80.00. Find the optimum run size and the minimum average yearly cost.
Solution
In this the demand rate (R) =600 per unit per year
 C1 = Holding cost per item per unit time = Rs. 0.60 per unit per year
 C3 = Set up cost per production run = Rs. 80 per production run

 ���������� Optimum lot size is   

� 

����������� = 0.67 years = 8 months


Thus the manufacturer should produce 400 units of his product at an interval of 8 months.
Example 2 A company uses 3000 units of a product, its carrying cost is 30%of average
inventory. Ordering cost is Rs. 100 per order. Unit cost is Rs. 20.Calculate EOQ and the total
cost.
Solution
D = Total Demand=3000 units
C1 = carrying cost =30% of Rs. 20=Rs.6
C3 = Ordering cost =Rs.100

 Optimum lot size is 


The total cost is equal to
Total cost = Material cost + Total variable cost
Jayaramulu123@gmail.com Page 25
����������� 
Operation Research

����������� 
11.6 Limitations of EOQ Formulae
In spite of several assumptions made in the derivation of above EOQ formulae, following are
the limitations while considering applications of these formulae:

i)     In the EOQ model we assumed that the demand for the item under consideration is
constant, while in practical situations demand is neither known with certainty nor it is
uniform.

ii)   It is difficult to measure the ordering cost and also it is not linearly related to number
of orders.

iii)  In EOQ model it is assumed that the annual demand can be estimated in advance
which is just a guess in practice.

iv)  In EOQ model it is assumed that the entire inventory which is ordered arrives
simultaneously. In many situations it may not be true.

v)    In EOQ, it is assumed that the demand is uniform; which may not hold in practice.

vi)   In EOQ model, the replenishment time is assumed to be zero which is not possible in
real life always.

Jayaramulu123@gmail.com Page 26
Operation Research

Jayaramulu123@gmail.com Page 27
Operation Research

Jayaramulu123@gmail.com Page 28
Operation Research

Jayaramulu123@gmail.com Page 29
Operation Research

Jayaramulu123@gmail.com Page 30
Operation Research

Jayaramulu123@gmail.com Page 31
Operation Research

Jayaramulu123@gmail.com Page 32
Operation Research

Jayaramulu123@gmail.com Page 33
Operation Research

Jayaramulu123@gmail.com Page 34
Operation Research

Jayaramulu123@gmail.com Page 35
Operation Research

Jayaramulu123@gmail.com Page 36
Operation Research

Jayaramulu123@gmail.com Page 37
Operation Research

Jayaramulu123@gmail.com Page 38
Operation Research

Jayaramulu123@gmail.com Page 39
Operation Research

Jayaramulu123@gmail.com Page 40
Operation Research

Jayaramulu123@gmail.com Page 41

You might also like